SBA Questions Flashcards

1
Q

A healthy year old man , 30, progressive severe retrosternal pain improved sitting foreword with a pericardial friction rub what would you see on the ECG?

A

Saddle shaped ST segment - Pericarditis

How well did you know this?
1
Not at all
2
3
4
5
Perfectly
2
Q

61 year old has exertion all chest tightness relived by rest - first degree heart block what will the PR interval be like

A

PR interval is greater than 200ms

How well did you know this?
1
Not at all
2
3
4
5
Perfectly
3
Q

43 year old African man has hypertension and diabetes mellitus whats the First line treatment

A

Ramipril - Always with diabetes give an ACE - I

normally African would give CCB - amlodipine

How well did you know this?
1
Not at all
2
3
4
5
Perfectly
4
Q

Asthma , hypertension, 67, tachycardia and irregularly irregular rhythm ( AF ) , high heart rate . BP and low resp rate how would you treat his hypertension and provide rate control

A

First line is beta blocker but if they are asthmatic you cant so give Verapamil ( CCB )

How well did you know this?
1
Not at all
2
3
4
5
Perfectly
5
Q

78 year old has hypertension and - Progressive lengthening of the PR interval with a dropped QRS what is this?

A

Second degree heart block : Mobitz 1 or wenckerbach pehnomenon

How well did you know this?
1
Not at all
2
3
4
5
Perfectly
6
Q

50 year old has leg pain on exertion and cramping in right calf when walking , worse if on an incline what is the first line Investigation?

A

ABPI and Duplex Ultrasound - Intermittent claudication of Periphral vascular disease ! Not DVT can be caused by atherosclerosis and 6 Ps : Pain, Pallor, pulselessnes,s parastehsia, paralysis, cold
First line is lifestyle , second is revascularisation , 3 is amputation

How well did you know this?
1
Not at all
2
3
4
5
Perfectly
7
Q

78, Fiti, Unwell, dizzy nausea , smoking, T2DM , no chest pain, hypotensive and ST elevation what blood test would you do?

A

Troponin - Atypical MI

How well did you know this?
1
Not at all
2
3
4
5
Perfectly
8
Q

35 year old man A and E with palpitations SOB, dizzy, chest pain , narrow complex tachycardia and SVT, valsalva and carotid sinus massage don’t work what do you do

A

Cardio version with adenosine - In shock straight to cardio version

How well did you know this?
1
Not at all
2
3
4
5
Perfectly
9
Q

What is a major complication of ACE inhibitors

A

HyperKalaemia

How well did you know this?
1
Not at all
2
3
4
5
Perfectly
10
Q

What does hyperkalemia show

A

Tall Tented T waves

How well did you know this?
1
Not at all
2
3
4
5
Perfectly
11
Q

Hypokalaemia

A

U waves, ST depression , T wav inversion

How well did you know this?
1
Not at all
2
3
4
5
Perfectly
12
Q

Mitral valve prolapse, low grade fever chills , fatigue , purple lesions on hand , dental surgery what investigation would you do ? What organism

A

Blood culture - Viridans Streptococci ( dental root canal )

Any person - Staph aureus

How well did you know this?
1
Not at all
2
3
4
5
Perfectly
13
Q

59 year old man comes to A and E with tearing chest pain , High BP. Absent pulse whats the gold standard investigation

A

CT angiography : Aortic Dissection - false lumen

First like USS may be if your desperate

How well did you know this?
1
Not at all
2
3
4
5
Perfectly
14
Q

18 feel unconscious swimming, congenital prolonged QT - what abnormal heart rhythm is he at risk of ?

A

Torsades de pointes - long pause on ECG between QRS and T wave ( long repolarisation time )
TDP is like the AM wave !!

How well did you know this?
1
Not at all
2
3
4
5
Perfectly
15
Q

65 has SOB acute , left sided chest pain worsened, bed bound , ECG shows regular Raytheon with R wave in V1 and slurred S wave in V 6 what is it showing ?

A

Right bundle branch block - William and Marrow acronyms
Only look at V1 and V6
Right loos like M =in V1 and W in V 6
William: W in V1 and M in V6

R wave: Bit going up (M) slurred S ( W) - RBBB

How well did you know this?
1
Not at all
2
3
4
5
Perfectly
16
Q

What is the underlying cause of the RBBB?

A

Pulmonary Embolism - Acute SOB, pleuritic chest pain, bed bound, recent surgery

How well did you know this?
1
Not at all
2
3
4
5
Perfectly
17
Q

Infant with Trisomy 31 has a pan-systolic Murmer at the left parasternal border whats the diagnosis

A

Ventricular septal defect: Downs most commen congential hear is VSD - 30%

TOF - 5%

How well did you know this?
1
Not at all
2
3
4
5
Perfectly
18
Q

What causes Rheumatic Fever?

A

Streptococcus Pyogenes - autoimmune condition and antibodies against it targets tissue - molecular mimicry - valvular damage

How well did you know this?
1
Not at all
2
3
4
5
Perfectly
19
Q

A 79 year old gentlemen has palpitations and lightheaded - go away , 120 bpm but irregular no clear P waves or ischaemia whats first line treatment ?

A

Metoprolol - haemodynamic ally stable and paroxysmal atrial fibrillation - use beta blocker if contraindicated digoxin or amlodipine, amiodarone is ventricualr tachycardias

How well did you know this?
1
Not at all
2
3
4
5
Perfectly
20
Q

30 has mild SOB, ejection systolic radiating to carotids what is it ?

A

Aortic Stenosis - ejection systolic loudest in aortic radiates to carotids
Mitral regurgitation - pan systolic radiating to the axilla

How well did you know this?
1
Not at all
2
3
4
5
Perfectly
21
Q

65 has chest pain radiation and ST elevation whats the immediate management ?

A

Morphine, aspirin, oxygen , GTN ( dont give atenolol

MONA : Morphine, oxygen, nitroglycerin, aspirin

How well did you know this?
1
Not at all
2
3
4
5
Perfectly
22
Q

79 crushing chest pain abnormalities in leads 2,3, and avF which artery is occluded ?

A

Right coronary artery - inferior leads

1, AVL and V 5 os LCx
V1-V4 is LAD

How well did you know this?
1
Not at all
2
3
4
5
Perfectly
23
Q

Cyanotic one month baby failure to thrive - TOF includes what defects :

A

Overriding aorta
Pulmonary valve stenosis
VSD
Right Ventricular hypertrophy

How well did you know this?
1
Not at all
2
3
4
5
Perfectly
24
Q

SOB worse on lying down , snores, stops breathing at night, peripheral oedema, basal crackles and raised JVP whats the first line to manage fluid overload?

A

Furosemide ( Loop diuretic ) - Congestive cardiac failure
Thiazide diuretics and aldosterone antagonists are second line
Other first lines are ramipril and bisoprolol

How well did you know this?
1
Not at all
2
3
4
5
Perfectly
25
Q

66 attends wellness check a GP , 30bpm no SOB, fit and well , hypertension and bronchiectasis , ECG shows 3 degree heart block what should you do?

A

No associations between P waves and QRS association , regular rate no connection - 3 degree - cardiac emergency - send directly to A and E - pacemaker

Escape rytham - bizarre QRS - wider

How well did you know this?
1
Not at all
2
3
4
5
Perfectly
26
Q

How does apixaban work?

A

Inhibits Factor Xa - DOAC
Warfarin is 1972 via vitamin K
Heparin inhabits thrombin

How well did you know this?
1
Not at all
2
3
4
5
Perfectly
27
Q

28 year old female has palps and dizziness - no medical conditions however drinks 47 units of alcohol and cocaine - saw tooth ECG , no P waves given diagnosis what is the definitive treatment ?

A

Atrial Flutter is saw tooth - definitive is catheter ablation of the circuit
BB are short term

How well did you know this?
1
Not at all
2
3
4
5
Perfectly
28
Q

81 collapsed , blacked out , light headed on standing , HTN, IHD, BPH and ex smoker, OD, bisoprolol, Ramipril, amlodipine, Bentroflumethiazie, tamsulosin all normal = but low sodium what caused the collpase?

A

Postural hypertension secondary to medications - tamsulosin and anti hypertensives - postural hypotension

How well did you know this?
1
Not at all
2
3
4
5
Perfectly
29
Q

Which is considered in CHAD3DS2-VSC ?

A
Age greater than 75 
Female Sex
History of CCF 
Previous stroke / TIA 
Hypertension 
DM 
Stroke/ TIA/ thromboembolism 
Vascular disease 
65-74 
Sex
How well did you know this?
1
Not at all
2
3
4
5
Perfectly
30
Q

23 has an ECHO after father passed away on a bike ride - HOCM what can you see on the ECHO?

A

Diastolic vetnricualar dysfunction ) LVhypertrophy, thickened septum, obstructs aortic valve

How well did you know this?
1
Not at all
2
3
4
5
Perfectly
31
Q

Exertion also dyspnoea, cough with pink frothy sputum perform a history and examination - heart failure

List some clinical signs of heart failure

What are the possible X ray findings

What blood test would confirm

How would you manage it

A
  1. Peripharal oedema, Orthopnea ( increased venous return - more oedema ) Paroxysmal nocturnal dyspnoea , bibasal creptiations
  2. Alveolar oedema, kerley B lines, Pleural effusion , cardiomegaly , dilated upper lobe vessel ( upper lobe diversion , ABCDE
  3. BNP N-terminal -pro-B Type natureitic peptide )
  4. Bisoprolol, Ramipril and furosemide : ACEI, Beta blocker , aldosterone antagonsit and loop diuretics ABAL
How well did you know this?
1
Not at all
2
3
4
5
Perfectly
32
Q

What is the difference between essentials and secondary hypertension

What Lifestyle advice would you offer a patient to reduce blood pressure

Name the most common cause of secondary hypertension

What scoring system assess risk of CVD disease

What type of medication for a score greater than 10%

Name a common side effect of this medication

A

Essential has no identifiable cause and secondary has one

Lifestyle: reduce salt / alcohol / caffeine, healthy balanced diet, regular exercise stop smoking

Commonest : Conns or Primary hyperaldosteronsism

QRISK score would then give a statin ( myalgia is the side effects ) - 10 years ( N and V)

How well did you know this?
1
Not at all
2
3
4
5
Perfectly
33
Q

55 Y/O post surgery coughing up blood streaked sputum and sharp unilateral chest pain worse on inspiration : BMI 30 and smokes 20 , high rate and resp rate:

What’s the diagnosis

What scoring tool would assess risk of her condition

List some risk factors

What’s the gold standard investigation and management

A

Pulmonary embolism - blood sputum, unilateral chest pain worse when breathing in

Wells score

Age, trauma, pregnancy, surgey , obesity, oestrogen,smoking

CT pulmonary angiogram and then treat with LMW heparin and warfarin
First line is Doppler - dual anticoagulant

How well did you know this?
1
Not at all
2
3
4
5
Perfectly
34
Q

28, chest pain, aching muscles and joints, sweaty and pale, bruising and track, small main fuel nodules ( Oslers nodes ) high tempurature normal ECG, haemturia

What is it ?

Which Pathogen ?

What is the next investigation

Gold standard investigation?

A

Infective endocarditis - Staphylococcus aureus, blood cultures, transesophageal echo

How well did you know this?
1
Not at all
2
3
4
5
Perfectly
35
Q

65 year old ongoing tight pain that comes and goes, raised BMI, sat down at rest, 2o smoking an HTN meds: ECG shows prinzmetals

How do you distinguish prinzmetals from unstable.

Give 2 non modifiable and 3 modifiable risk factors for angina

What lifestyle Advice would you give

What pharmacological Mangement

A

ST elevation in prinzmetals - transmural ischaemia in unstable causes ST depression due to subendocardial ischaemia

2 Non modifiable : Age, FH and ethnicity ( Angina is more commen in females MI in males)
Modifable: BMI, Smoking , diet , inactivity, stress, HTN, cholesterol and diabetes

Weight loss via exercise and diet and smoking cessation

First line : CCB e.g. Verapamil all Anginas get GTN

How well did you know this?
1
Not at all
2
3
4
5
Perfectly
36
Q

A 23 year old lady has unintentionally lost 8kg of weight over the last 8 months and is suffering from episodes of fatigue. She also complains of nausea, headaches, generalised abdominal pain and is feeling terrible overall. How would her cortisol levels react to the Synacthen test if she had secondary adrenal insufficiency?

A

Short ACTH no change long ACTH increase

-Synacthen stimulates the adrenal glands to secrete corticosteroids in secondary adrenal insufficiency

How well did you know this?
1
Not at all
2
3
4
5
Perfectly
37
Q

A deficit in which hormone in this patient’s condition has caused the development of hyperpigmentation?

A

ACTH - POMC proopiomelanocortin

How well did you know this?
1
Not at all
2
3
4
5
Perfectly
38
Q

Where do carcinoid tumours usually metastasise to>

A

Liver! - neuroendocrine tumours

How well did you know this?
1
Not at all
2
3
4
5
Perfectly
39
Q

A plasma aldosterone/renin ratio is performed and a diagnosis of primary hyperaldosteronism is made. Which electrolyte imbalances are associated with this condition?

A

Hypokalemia and hypernatremia

To much aldosterone - increases sodium reabsorption and water follows sodium - high sodium and hypertension and potassium is drawn out

Hypokalemia@ Weakness, flaccid paralysis and hyporeflexia
High aldosterone and low renin

How well did you know this?
1
Not at all
2
3
4
5
Perfectly
40
Q

With PTH, calcium and phosphate levels would you expect to find with someone with hyperparathyroidism caused by renal disease?

A

High PTH, low calcium and high phosphate - hypocalcaemia due to decreases 1-25 dihydroxycholecalciferol levels - more PTH and high phosphate as kidneys cant excrete it

How well did you know this?
1
Not at all
2
3
4
5
Perfectly
41
Q

A 54 year old man with known DMT2 presents into the clinic complaining that his medications are not working. He is still presenting with symptoms despite compliance to medication. However, recently he has developed a loss of appetite and an upset stomach. After taking some investigations the patient is said to have lactic acidosis with severe liver failure.

He is currently on triple therapy for his diabetes taking metformin + sulfonylurea + SGLT-2 inhibitor (glifazone), as well as, Ramipril for his hypertension.

A

Metformin: Increases sensitivity, causes weight loss but GI disturbances, peripheral neuropathy and lactic acidosis

Sulphonylureas: Increase insulin secretion in B cells but GI disturbances, hypoglycaemia and weight gain

Pioglitazone: Reduces reistance but bone fractures , weight gain and bladder cancer and is contraindicated in HF and bladder cancer

DPP-4: Increase DPP-4 to increase insulin and lower glucagon - headache and acute pancreatitis but is cardioprotective

SGLT-2 Inhibitors: Inhbit cotransporter to reduce glucose reabsoprtion and increase excretion of glucose but can cause UTI, general pruritis , DKA and back pain - cardioprotective but CI in T2DM

How well did you know this?
1
Not at all
2
3
4
5
Perfectly
42
Q

A 48 year old lady presents to your surgery complaining of numbness and tingling in her right hand. She also appears to have large hands, rough/tanned skin and a prominent jaw and forehead. None of her clothes fit as she has also gained 10kg in the last year. Which nerve roots are associated with the condition presented above?

A

Carpal tunnel syndrome: C5/6-T1 median nerve of the medial and lateral brachial plexi cords

How well did you know this?
1
Not at all
2
3
4
5
Perfectly
43
Q

A 45 year old woman presents to her GP complaining of feeling unsteady. She has mentioned that she has become increasingly more sweaty recently despite it being the winter months. Along with this she is saying that she has begun to eat more than usual. But what’s confused her most is that despite this, she is still losing weight.

However, what’s been most worrying for her is that she has begun to develop what seems to be waxy discoloration to her lower legs which she describes as orange peel appearance.

Which antibody is most likely to be associated with the condition presented above?

A

Graves - TSH receptor antibodies

Hashimotos - Anti TPO

Anti DsDNA- SLE

Anti thyroglobulin is hashimotos

Graves triad: Goitre , orbitopathy, pretibial myxoedema

How well did you know this?
1
Not at all
2
3
4
5
Perfectly
44
Q

A 15 year old boy with poorly controlled type 1 diabetes comes to A&E with a 2 day history of abdominal pain, vomiting and fruity smelling breath. What is the diagnostic criteria for his suspected condition?

A

Ketones >= 3 mmol/L, glucose > 11 mmol/L and pH < 7.3

How well did you know this?
1
Not at all
2
3
4
5
Perfectly
45
Q

A 52 year old male has noticed that his skin is becoming coarser and his shoes no longer fit. Looking over past pictures he has noticed overall enlargement in peripheral limbs. He was initially referred for surgery but is no longer fit enough to undergo the procedure as he is suffering from heart failure as a result of this condition. What is the next best management for him regarding this condition (not heart failure)?

A

Somatostatin analogue

1st line - transsphenoidal surgery
2nd line - Somatostatin analogue (SSA)
octreotide or lanreotide
SE: pain at injection site, abdo cramps, flatulance, loose stools, ↑ gallstones, impaired glucose tolerance
\+/- Dopamine agonist if GH secretions persist
3rd line - GH-receptor antagonist (GHRA)
pegvisomant
4th line - Radiotherapy
How well did you know this?
1
Not at all
2
3
4
5
Perfectly
46
Q

An elderly gentleman is difficult to rouse and appears less responsive than usual. He is currently recovering from a chest infection in hospital and has a patent cannula for medications/fluids. Significant PMH includes type 2 diabetes and ischaemic stroke which caused him to become NBM. His blood glucose is currently 2.3 mmol/L. What is the best management for this patient?

A

IV glucose - NBM so oral glucose is not an option and there is no point giving glucagon because they dont have enough glucose stores

How well did you know this?
1
Not at all
2
3
4
5
Perfectly
47
Q

A young gentleman presents to the GP practice complaining of sweating more than usual. Under further investigation he tells you that he has also noticed more frequent headaches and a feeling of his heart pounding his chest. He tells you that this has gotten him more anxious recently as this has never happened before.

Given the diagnosis, what is the mechanism of action for the first line treatment given to this patient?

A

Blocks the alpha-adrenergic receptors leading to vasodilation
1st line in hypertensive crisis - phentolamine

2nd line is sodium nitroprusside

How well did you know this?
1
Not at all
2
3
4
5
Perfectly
48
Q

Hyper / Hypo hypothyroidism

What is the difference in symptoms between the two? List 5 for Hyper and 5 for hypothyroidism.

How does first line management differ in Hyperthyroidism + Hypothyroidism?

What is the most common cause of hypothyroidism in the developing world?

A

Hyper: Sweating, heat intolerance, weight loss, manic restlessness, palpitations, preorbital myxoedema, erythematous oedematous lesions, thyroid acropachy , diarrhoea, oligomenorrhea , anxiety and tremor

First line: Propranolol, carbimazole and iodine

Hypo: weight gain , lethargy , dry cold, yellow skin, non pitting oedema, dry coarse scalp, constipation, menorrhagia, carpal tunnel and decreased tendon reflexes

1st line is levothyroxine

Developing : iodine deficiency and hashimotos thyroditis

How well did you know this?
1
Not at all
2
3
4
5
Perfectly
49
Q

What glucose levels are seen in a non diabetic, pre-diabetic and diabetic?

Describe the management plan in a patient with newly diagnosed diabetes mellitus type 2 + what would be the follow up management if symptoms progress?

What are some complications of DM?

A

HBA1c less than 42, then 42-47 then 48 and over
Lifestyle modifications - Diet, Exercise, Weight loss
Monotherapy - Metformin
Dual therapy - if HbA1c rises to 58mmol/mol (7.5%)
Triple therapy - if HbA1c rises to 58mmol/mol (7.5%)
Metformin
DPP4 inhibitor
SU, Pioglitazone
SGLT-2 inhibitor
Thiazolidinedione
insulin based therapy ONLY WHEN MEDS FAIL
Insulin or Glucagon-like peptide (GLP) analogues
GLP analogues - incretin mimetics
Incretins - gut peptides that work by increasing insulin release.

If 58 mmo/L / 7.5% or higher then step up the treatment

Acute - DKA, HHS, hypoglycaemia
Chronic
Microvascular - retinopathy, nephropathy, erectile dysfunction, neuropathy (10-20 years after diagnosis in young patients)
Macrovascular - atherosclerosis, ACS, stable angina, PVD, stroke
Diabetic foot and ulcers

How well did you know this?
1
Not at all
2
3
4
5
Perfectly
50
Q

Cushing’s syndrome and Cushing’s disease

What is the difference between Cushing’s disease and Cushing’s syndrome?

Which investigation can be used to differentiate between the two conditions?

How does a patient with Cushing’s syndrome present (5 marks)?

A

Cushing’s syndrome refers to the state of elevated cortisol levels
Cushing’s disease is specifically caused by an ACTH secreting pituitary tumour (pituitary adenoma)

Dexamethasone suppression test
Overnight
Cushing’s syndrome (including disease) is confirmed when there is no suppression
48 hour
Cushing’s syndrome (not including disease) = no suppression

Obesity, buffalo hump,purpura, abdominal striae, muscle weakness, poor wound healing, hypertension, moon facies, thin skin, amenorrhea,

How well did you know this?
1
Not at all
2
3
4
5
Perfectly
51
Q

You are an F1 on the endocrine ward and your consultant shows you lots of different ranges of TSH and T3/T4 levels and asks you to interpret the results.

Which conditions would most likely cause the following TSH and T3/T4 results?

You are an F1 on the endocrine ward and your consultant shows you lots of different ranges of TSH and T3/T4 levels and asks you to interpret the results.

TSH: Low T4: High

TSH: High T4: Normal

TSH: High T4: Low

TSH: Low T4: Low

A
  1. Primary hyperthyroidism e.g. graves disease, thyrotoxicosis
  2. Poor thyroxine compliance or subclinical hypothyroidism
  3. Primary hypothyroidism e.g. hashimotos
  4. Secondary hypothyroidism
How well did you know this?
1
Not at all
2
3
4
5
Perfectly
52
Q

A patient comes into the Hospital with suspected DKA.

What are the first line treatments you would offer?

What ecg changes are seen in a patient with hyperkalemia (list 4)?

What arrhythmia is a consequence of untreated hyperkalaemia?

A

A patient comes into the Hospital with suspected DKA.

What are the first line treatments you would offer?
IV Fluids (0.9% Sodium Chloride)
IV insulin +/- Potssium

What ecg changes are seen in a patient with hyperkalemia (list 4)?
Go - Absent p waves
Go Long - prolonged PR
Go Tall - Tall T waves
Go Wide - Wide QRS
‘Sine’ Wave appearance

What arrhythmia is a consequence of untreated hyperkalaemia?
Ventricular Tachycardia
Occurring due to cell membranes becoming partially depolarised resulting in a lower threshold potential -> Ventricles contract quicker

How well did you know this?
1
Not at all
2
3
4
5
Perfectly
53
Q

Shane is 34yr old male with Down’s syndrome. He has recently been complaining of increasing breathlessness and fatigue over the last 3 or 4 weeks. Upon looking at his medical record, you also notice he has been getting infections much more commonly than normal.

On examination you note bruising on his legs but otherwise normal. Which of the following are you most likely to see on a peripheral blood film?

A

Auer Rods

Bone marrow infiltration and failure will lead to:
Bruising / bleeding - thrombocytopenia
Infections - leukopenia 
Breathless, fatigue, pale - Anaemia
(pancytopenia if all 3 are present)

Acute myeloid leukaemia: Age points to AML , ALL is younger and CNS - smudge cells

How well did you know this?
1
Not at all
2
3
4
5
Perfectly
54
Q

Katie, a 25-year old student has been struggling with tiredness for a few months. She presented to the GP having developed weakness and pins and needles in her hands and feet, and difficulty with balance and walking. She has been following a vegan diet for the past 7 years.

What blood tests would you carry out and what would you expect the results to be? (3 marks)

A

What blood tests would you like to carry out and what would the likely results be? (3 marks)
FBC
↓Hb
↑MCV (macrocytic anaemia)
Blood film
Hypersegmented neutrophils (>5 lobes) and presence of oval macrocytes
Serum cobalamin ↓

How well did you know this?
1
Not at all
2
3
4
5
Perfectly
55
Q

6 month baby A and E high fever and rashes that don’t disappear , unwell in pain what is it

A

Meningitis - non blanching rashes indicative of meningococcal infection and fever

GB: Weakness of legs and arms, campylobacter - cant close eyes
Encephalitis: Weakness and sleepiness
Pneumonia: Non productive cough and breathing difficulties
IE: Fever and new murmer

How well did you know this?
1
Not at all
2
3
4
5
Perfectly
56
Q

Alice has meningitis causes by a fungus infection what findings on LP

A

Bacterial : Granulocytes, protien high and glucose low
TB and Fungal: Lymphocytes present protien high glucose low
Viral lymphocytes , protien high and glucose normal

How well did you know this?
1
Not at all
2
3
4
5
Perfectly
57
Q

19 and headache that comes and goes , studying for exams around whole forehead but no cough or cold or fever ?

A

Tension headaches - bilateral pain behind eyes, under stress, low sleep
Cluster: Headache is unilateral with pain in episodes, rhinorrhea and tears
Migraines: Unilateral puking photo, sound phobia and aura
Sinusitis: bilateral hurts in all, fever and sickness
Temporal Aura: Sharp unilateral pain that comes for a few seconds that goes away when touching or shaving face

How well did you know this?
1
Not at all
2
3
4
5
Perfectly
58
Q

23 unilateral headache 3 years comes and goes, weird zig zag lines before getting headaches, father has it to and asthma what is the prophylactic treatment?

A

Migraine with aura - normally beta blockers but asthmatic so topiramate ( anti epileptics )( not in pregnancy ) - contraceptive

Verapamil is used for cluster
Sumatriptan is first line for migraines with an NSAID
Amitriptyline used if both cant be

How well did you know this?
1
Not at all
2
3
4
5
Perfectly
59
Q

Mary has severe pain on one side of her head that comes and goes very tender and intermittent jaw claudication ?

A

It is Giant cell arteritis - jaw claudication and severe pain
First line is CRP and ESR
Gold standard is Temporal artery biopsy
Treat with prednisolone - vessel Inflammation
Can causes vision loss, stroke , blindness ( temporal close to optic )

How well did you know this?
1
Not at all
2
3
4
5
Perfectly
60
Q

Sarah is presenting to A and E with a seizure after collapsing , lost conciousness, 12 weeks pregnant diazepam and lorazepam didn’t help - arms and legs flailing and jaw clenching ?

A

Generalised tonic clonic seizure - failing and clenching more then 5 minutes its now become status epilepticus
Because lorazepam didn’t work your then you give lorazepam again then phenytoin
Epilepsy is seizures of unknown cause she is pregnancy so it may be pre eclampsia
2 or more 24 hours apart and Eliminate all causes e.g. electrolyte imbalances
If they didn’t have epilepsy lamotrigine - pregnancy

How well did you know this?
1
Not at all
2
3
4
5
Perfectly
61
Q

Weird jerks, all the time , lost structure -19 year old boy

A

Focal seizures - Jacksonian march - arms to back - to leg

To young for Huntington’s chorea - 40 and no FH and no behavioural change
petit mal - absence seizures, staring loss of awareness

How well did you know this?
1
Not at all
2
3
4
5
Perfectly
62
Q

Jack is 59 - stroke like symptoms - headaches, feeling dizzy, nauseous, homonymous heminapoia , weak , HT, Hcholestroal and diabetes what is first line investigation ?

A

Order CT scan - positive for Ischaemic stroke - IV alteplase can be given within 4.5 hours so use aspirin and mechanical thrombectomy ( 6-24)
Posterior stroke but if it was double vision also posterior circulation - lateral Rectus and superior oblique
Amaurosis fugax
Check to make sure its not haemorrhagic

How well did you know this?
1
Not at all
2
3
4
5
Perfectly
63
Q

72 year old man has unusual hemiparesis cant life arm and leg but can feel what is he having:

A

Lacunar stroke - motor not sensory function - penetrating arteries not main

Stroke aetiology: AF, atherothromboembolism of carotid

Strokes are either :
Total or partial  anterior circulation 
Posterior 
TIA 
Lacunar
How well did you know this?
1
Not at all
2
3
4
5
Perfectly
64
Q

Sudden onset headache, soreness around neck and shoulders and photophobia whats the next step in management worst in first 3minutes

A

Subarachnoid haemorrhage - severest pain in first 3minutes then gets not as bad - CT scan
This is Menigism : headache, photophobia and neck stiffness ( SAH and Menigitis , migraines ) LP 12 hours after onset to allow xanthochromia
Migraine : Mild to moderate is paracetomal and high is rescuer with sumatriptan

What are complications of menigism:

  • Hydrocephalus and vasospasm - nimodipine
  • seizures, rebleeding and electrolyte imbalances
How well did you know this?
1
Not at all
2
3
4
5
Perfectly
65
Q

67 has confusion in A and E with traumatic flow to the head , recovered quickly, past medical history, high cholesterol, crescent shaped haaematoma on the left with midline shift what is the most common cause and 4 risk factors:

A

Subdural haemorrhage - rupture of the bridging veins
- hypertension, trauma, age, alcohol, infant abuse shaken babies, and anticoagulants
Acute - younger
Chronic in olde r-n atrophy stretches bridging veins - ruptured due to trauma

How well did you know this?
1
Not at all
2
3
4
5
Perfectly
66
Q

Joe is 59 detained under mental health act, acting strangely , rude comments, addenbrookes normal but atrophy of the right frontal lobe what is the diagnosis ?

A

Acting weird and histopatholhou of Alzheimer’s: Neurofibrillay tangles, beta amyloid s, atrophy , loss of ACH productions

How well did you know this?
1
Not at all
2
3
4
5
Perfectly
67
Q

Jane is 30 with acute pain onset in left eye, seeing double cant tell colours apart, more tired, dragging foot, lesions in white matter? How would you manage

A

Multiple Sclerosis - optic neuritis :2 episodes disseminated in time and space
Methylpredisalone - gabapentin is for pain
Propranolol

How well did you know this?
1
Not at all
2
3
4
5
Perfectly
68
Q

Types of MS

A

Relapsing remitting,progressive primary and secondary progressive

How well did you know this?
1
Not at all
2
3
4
5
Perfectly
69
Q

4 cardinal brain tumours Sympotyms?

A

Raised ICP, focal neurological deficit. visual disturbances, seizures, lethargy

Lung tumour can metastases to : Brain liver adrenal and bone
Lung tumours met from: Bowery, bladder, breast, renal cell carcinoma
Name 5 places prostate can golf L prostate, kidney, breast and bone

How well did you know this?
1
Not at all
2
3
4
5
Perfectly
70
Q

43 year old has progressive difficulty walking and lower back pain, tripping and cant climb stairs, tingling and numbness , food poisoning , what is the most important parameter to moniter?

A

Guillian Barre syndrome - gastroenteritis , ascending polyneuropathy
FVCapacityb - monitor pulmonary function because of ascending neuropathy to diaphragm - failure

How well did you know this?
1
Not at all
2
3
4
5
Perfectly
71
Q

A 40 year old is referred to a neurology clinic - double and blurred vision more tired no abnormalities which of the following tests is she positive for?

A

Myasthenia gravis - gets worse towards end of day - antibodies to ACH receptors
If its negative to Anti - MUSK

Bedside test: ICE PACK test and upgaze
Scan - Thymus CT
Lambert Eaton - SCC - antibodies to pre synaptic voltage gated calcium channels
Not MS no pain

How well did you know this?
1
Not at all
2
3
4
5
Perfectly
72
Q

A 28 year old construction worker has night numbness and tingling , shaking hands relives symptoyms, no weakness of ahnds - affects thumb , index and middle fingers, no wasting , wore normal, tapping the wrist with a tendon hammer reproduces symptoms - tinnels test

A

Median neuropathy and carpal tunnel syndrome - construction worker and afects thumb, index and half of middle - median nerve
Tinnels positive and phalens test

Aetiology: Idiopathic, pregnancy, high BMI, wrist fractures, RA, severe oedema and HF, occupation
Electrophysiology - prolongation of action potential - EMG
TX: Wrist splints 1st line in mild or pregnancy
Corticosteroid injections
Surgical related if severe, non pregnant - definitive

Median nerve - flexion loss rock
Radial - extension loss paper
Ulnar - finger abduction and abduction loss scissors

How well did you know this?
1
Not at all
2
3
4
5
Perfectly
73
Q

69 presents with back pain and dragging right foot, Dorsiflexion weakness, normal inversion and sensory loss of medial aspect whats the cause?

A

Foot drop - emergency is cauda equine but most common is common peroneal nerve palsy and L5 rediculopathy
In spinal nerve compression - lose sphincter and bladder control - high storage gait
Common peritoneal never e- lose version and sensory loss due to compression a fibula neck e.g. leg crossings , kneeling bakers cyst and platers casts

How well did you know this?
1
Not at all
2
3
4
5
Perfectly
74
Q

An 84 year old man has a pill rolling tremor at rest:
What imaging would confirm diagnosis and what do your expect to see
What lesion in the brain
Apart from tremor give 2 other clinical Parkinsons hallmark
Name 2 other causes of Parkinsonism
Name 3 medications

What about if tremor when anxious relived by alcohol

A

Parkinsons
Use DATscan - substantia nigra - degradation and degeneration
MRI lesion would be Lewy bodies
Tremor, bradykinesia and rigidity
Multi - system atrophy, drug inducesd, progressive supranucleur palsy and cortico-basal degeneration
Medications: Levodopa ( Dyskinesia , involuntary movement, falls , postural hypertension) Dopamine agonist - bromocriptine , bropinarol ( pulmonary fibrosis, increase gambling and impulsivity )MOA_-B inhibtors

Benign or essential listening tremor

How well did you know this?
1
Not at all
2
3
4
5
Perfectly
75
Q

You are a junior doctor in GP a 70 year old man is struggling to wee whats the first line?

A

Tamulosin - BPH its an alpha blocker which allows relaxation of the prostate muscle

How well did you know this?
1
Not at all
2
3
4
5
Perfectly
76
Q

Which arteries supply 80% of the blood to the brain

A

Internal Carotids - supply 80% with 20% being vertebral

How well did you know this?
1
Not at all
2
3
4
5
Perfectly
77
Q

A 16-year-old boy presents to A&E with severe pain in his left groin area that started when he was playing football with his friends. On examination, the left side of his scrotum is red and swollen and his cremasteric reflex was absent on the left. What is the diagnosis?

A

testicular torsion

Hydroceles, varicoceles, and epididymal cysts are normally painless. In someone with epididymitis the presentation is similar but, the cremasteric reflex would be present. The cremasteric reflex involves the testis moving upwards when the inner thigh is stroked.

How well did you know this?
1
Not at all
2
3
4
5
Perfectly
78
Q

What isnt a cause of ischaemic stroke: Vasculitis, embolism, thrombosis, carotid artery dissection, aneurysm rupture

A

Aneurysm rupture

How well did you know this?
1
Not at all
2
3
4
5
Perfectly
79
Q

A 52-year-old man presents after having another episode of severe pain down the left side of his abdomen. He previously had an episode a few days ago. He says the episodes don’t last very long but when they happen, he can’t get comfortable and describes the pain as the worst pain he has ever felt in his life. He says he sometimes feels sick when the pain happens. What would be the best investigation to perform?

A

This presentation is consistent with kidney stones. NCCT-KUB is the gold standard investigation for kidney stones.

How well did you know this?
1
Not at all
2
3
4
5
Perfectly
80
Q

An 81 year old lady atends your GP practce asking about her
risk of having a stroke. She has a past medical history of type
2 diabetes and hypertension and is on warfarin. Which of the
following factors does not increase her risk of having a
stroke?

A

Gender

NM: Male, older age, FH, ethnciity, APL
Modifable: HTN, DM, CVD, PVD, Hyperlipidaemia, excessive alcohol use

How well did you know this?
1
Not at all
2
3
4
5
Perfectly
81
Q

A 72-year-old man has recently been diagnosed with chronic kidney disease (CKD). Some blood tests have been done which show his glomerular filtration rate (GFR) is 48ml/min/1.73m2. Which stage of CKD is this?

A
GFR is how well the kidneys are able to filter blood and is an indication of kidney function. Normal GFR should be at least 90 which is stage 1. 
Stage 2 -> GFR between 60 and 80
Stage 3a -> GFR of 45-59
Stage 3b -> GFR of 30-44
Stage 4 -> GFR of 15-29
Stage 5 -> GFR of less than 15
How well did you know this?
1
Not at all
2
3
4
5
Perfectly
82
Q

A 21 year old student presents to A&E with reduced consciousness following a
head injury which he sustained afer getng in a fght at the pub several hours
ago. You take a history from his friend who has brought his to A&E.
His friend tells you that he did not lose consciousness afer hitng his head on
the ground but was drowsy and confused. He then improved and was okay for
a couple of hours apart from having a headache. He then started vomitng and
became unresponsive. What do you think is the most likely diagnosis?

A

Extradural haemorrhage - lucid interval

How well did you know this?
1
Not at all
2
3
4
5
Perfectly
83
Q

A 7 year old girl presents to the out of hours GP with fever,
headache and a rash. On examinaton she has a stf neck and keeps
covering her eyes to avoid looking at the bright lights in the clinic
room. What do you expect the rash to look like?

A

Non Blanching petechial rash

How well did you know this?
1
Not at all
2
3
4
5
Perfectly
84
Q

A 32 year old gentleman presents to your GP practce complaining
that his legs feel weak. On further questoning, the weakness
started in his ankles a couple of days ago but is now afectng the
whole of his legs. He says that he’s fnding it really hard to walk
upstairs and stand up from the sofa. He has no other symptoms. He
is normal ft and well and takes no regular medicaton. He had a bit
of a cold a few weeks ago but he tells you it wasn’t anything
serious. What is the most likely diagnosis?

A

Guillian Barre syndrome

How well did you know this?
1
Not at all
2
3
4
5
Perfectly
85
Q

A 78 year old woman atend your memory clinic with her daughter.
Her daughter tells you that her memory has been getng worse
over the past 12 months. Recently her neighbours have found her
wandering around the street late in the evening. She is otherwise
well and take no other medicaton apart from a multvitamin. What
is the most likely diagnosis?

A

Alzheimers

How well did you know this?
1
Not at all
2
3
4
5
Perfectly
86
Q

A 78 year old woman atend your memory clinic with her daughter.
Her daughter tells you that her memory has been getng worse
over the past 12 months. Recently her neighbours have found her
wandering around the street late in the evening. She is otherwise
well and take no other medicaton apart from a multvitamin. What
is the most likely diagnosis?

A
Cheese
Oral contraceptive
Caffiene
Anxiety 
Travel 
Exercise
How well did you know this?
1
Not at all
2
3
4
5
Perfectly
87
Q

a. Defne an ischaemic stroke. (2 marks)
b. Give 2 modifable and 2 non-modifable risk factors for a
stroke. (4 marks)
c.
You perform a neurological examinaton. Other than his
facial droop, list 2 other signs you may fnd? (2 marks)
d. What is the 1st line investgaton? (1 mark)
e. Your 1st line investgaton shows no sign of a haemorrhagic
stroke, what inital treatment would you give? (1 mark)

A

jckxbzbzlx

How well did you know this?
1
Not at all
2
3
4
5
Perfectly
88
Q

) zxxz

A

List 3 signs of Parkinson’s disease (3 marks)
Bradykinesia, rigidity, tremor, postural instability…
b.
Explain the pathophysiology of Parkinson’s disease (2 marks)
Progressive neurodegeneraton of dopaminergic neurons in the substanta
nigra resultng in dopamine defciency which results in reduced movement
initaton - increased inhibiton of the thalamus
c. Name 2 other diferental diagnosis. (2 marks)
benign essental tremor, lewy body dementa, drug induced parkinsonism,
stroke, PSP, MSA, Wilson’s disease
d. Give 1 pharmacological and 1 non-pharmacological aspect of PD management (2 marks)
Pharmacological: levodopa, dopamine agonists, MOAB inhibitors
Non-pharmacological: physio, OT, SALT, deep brain stmulaton
e.
Name 1 complicaton of Parkinson’s disease (1 mark)
Reduced quality of life, depression, treatment side efects, dementa,

constpaton
a

How well did you know this?
1
Not at all
2
3
4
5
Perfectly
89
Q

A 73 year old right-handed man presents with sudden onset weakness of his right arm and leg and slurred speech. He
has a history of ischaemic heart disease, diabetes and uncontrolled hypertension.
On examinaton, he is alert and is able to provide a history, although his speech is not normal. He has facial asymmetry,
and profound weakness of his right arm and leg. Non-contrast CT scan done within 40 minutes of symptom onset shows
no obvious acute pathology. Blockage of which artery is likely to be responsible for his syndrome?

A

Left middle cerebral

Anterior - personality and legs

How well did you know this?
1
Not at all
2
3
4
5
Perfectly
90
Q

21 girl seizures

A

Lamotrigine

Carbamazepine would be if he would

How well did you know this?
1
Not at all
2
3
4
5
Perfectly
91
Q

A 30 year old man is referred to the clinic with sensory loss on his litle fnger and the lateral half of his ring fnger. He
has weakness bending his fngers although he can bring his thumb up vertcally with good resistance. Refexes are
preserved. Which nerve has he injured?

A

Ulnar

Radial - base of thumb and back of hand

median is indez and middle and half of the ring on palmar sid e

How well did you know this?
1
Not at all
2
3
4
5
Perfectly
92
Q

A 72 year old woman is reviewed in neurology outpatents for ongoing management of her Parkinson’s disease.
She was diagnosed 3 years ago when she notced a subtle right side restng tremor and generalised slowness of
movement. She takes co-careldopa daily, and last year was started on entacapone to reduce her end-of-dose efect.
Recently she has notced the development of slow writhing movements of her upper limbs, most prominent in the
morning.
What is the underlying cause of the patent’s new symptoms?

A

Long term levodopa - writhing movements of the limsb

progression of parkinsons - ridgidty nad trenir

How well did you know this?
1
Not at all
2
3
4
5
Perfectly
93
Q

A 68 year old man with a history of hypertension obesity and type 2 diabetes presents with fatigue, swollen legs and itchiness GFR is 20 whats the diagnosis

A
Stage 4 - know CKD and AKI  - water retintion, fatigue, toxin metabolise build up ( urea ) 
1 - above 90 
2 - 60 - 89
3A: 45- 59
3b : 30-44
A 15-29
5 less then 15 renal failure
How well did you know this?
1
Not at all
2
3
4
5
Perfectly
94
Q

Lightheadeness and dizziness out of bed or office chair, fainted , what medication is doing this:

A

Tamulosin- BPH can cause postural hypertension

furosemide can cause it to but they are CI in CKD

How well did you know this?
1
Not at all
2
3
4
5
Perfectly
95
Q

50 year old man presents to A and E with a 1 hour history of sudden onset left sided flank pain radiating to groin - stabbing 10/10 nauseous and IV tiny

A

NCCT - KUB ( renal colic ) - sudden onset ipsilateral pain radiating down groin, loin to groin cant sit still
Gold standard - NCCT KUB - contrast can damage
Renal ultrasound younger and reoccurring
Watch and wait. Treat , tamsulosin, anti emetic , surgical innervation
KUB - X ray is first line but cant see pure Uric acid

How well did you know this?
1
Not at all
2
3
4
5
Perfectly
96
Q

76 PMS women has pain urinating and frequancy - has previous epsiodes which are treated with NFT:

A

E.Coli ( Klebsiella second )

How well did you know this?
1
Not at all
2
3
4
5
Perfectly
97
Q

Facial oedema - total body swelling, face , abdomen scrotum and fever had a viral illness and fever, heavy protienuria what is it - 5 year old?

A

Minimal change disease !
IgA is post tonsillitis an dis nephritic - haemturia and mimial protienuria
Hoskins - 20 or 75, painful lump after a night alcohol
Acute glomerulonephritis post strep adults

How well did you know this?
1
Not at all
2
3
4
5
Perfectly
98
Q

A 70 year old has puffy legs and fatigue - Periphral oedema but normal - antibiotics for infection which antibiituc causes AKI

A

Gentamicin

How well did you know this?
1
Not at all
2
3
4
5
Perfectly
99
Q

What can chloramphenicol do

A

Bone marrow suppression and amoxicillin can cause hypersensitivity

How well did you know this?
1
Not at all
2
3
4
5
Perfectly
100
Q

6 year old feels under weather - puffy barely pees, bacterial tonsillitis, protien and blood in the urine

A

Post streptococcal glomerulonephritis

IGA nephropathy = 2 days
Post strep is 1-3 weeks
Wilma Tumor - mass

How well did you know this?
1
Not at all
2
3
4
5
Perfectly
101
Q

50 year old - haemorrhagic stroke, well no fever or dysuria, recurrent UTI, FH of CKD and passed away from a stroke at 65

A

Renal USS- haemorrhagic stroke and CKD, recurrent UTIs, FH, PKD

How well did you know this?
1
Not at all
2
3
4
5
Perfectly
102
Q

75 year old has difficulty Uric ting and bone pain, prostate is hard and irregular , lost weight which part of the prostate is abnormal?

A

Peripharal zone becomes cancerous

Transitional zone is BPH

How well did you know this?
1
Not at all
2
3
4
5
Perfectly
103
Q

A 74 year old male has haemturia and and lethargy with background of schistosomiasis what kind of bladder cancer is it ?

A

Squamous cell carcinoma - schistosomiasis

Normally urethilial carcinoma

How well did you know this?
1
Not at all
2
3
4
5
Perfectly
104
Q

Which cancers spread to the bone

A
Kidney 
Thyroid 
Breast 
Prostate 
Lung
How well did you know this?
1
Not at all
2
3
4
5
Perfectly
105
Q

Modes of cancer transport

A

Haematological
Peritoneal
translimical
Lymph node

How well did you know this?
1
Not at all
2
3
4
5
Perfectly
106
Q

Worsening left scrotal pain and white urethral discharge, tender erythematosus swollen left scrotum, intact cremasteric reflex and relief with elevation

A

Epididymo - orhcitis

Cysts , hydrocele and tumour - painless lump
Testicular torsion is an acute emergency - phrens reflex negative

How well did you know this?
1
Not at all
2
3
4
5
Perfectly
107
Q

19 has dysuria - unprotected oral and veg animal intercourses , slight white dispatcher no tenderness - chlamydia what is the first line management?

A

Doxycycline

Not good in pregnancy - erythromycin or amoxicillin or axirthyromsyicb

How well did you know this?
1
Not at all
2
3
4
5
Perfectly
108
Q

34 otherwise healthy fever, haemturia, increased frequancy right sided back OAB, - pyelonephritis whats the classical triad

A

Nausea and vomiting, fever and loin /flank pain - E.coli , female, vesicles ureteric reflux
Cephalaxin
Take sample before MSU
Broad spectrum then specific

How well did you know this?
1
Not at all
2
3
4
5
Perfectly
109
Q

66 - urgency’s nd frequancy , 12 times a day and. Times a night she leaks urine occurs as she stands up which nerve contracts the detrusor

A

Pelvic nerve
Symapethic is hypogastric nerves fight and flight - relaxes ( T12 - L2)
Parasymapethic Pelvic S2-S4
Pudendal is sphincter somatic ( S2-S4

KNOW BRACHIAL PLEXUS, lower limb innervation ( mononeuropathy s )

How well did you know this?
1
Not at all
2
3
4
5
Perfectly
110
Q

35 non specific set half discomfort - 2cm by 1cm smooth painful mass, testicular cancer and referred to US what is associated with the diagnosis ?

A

Increased serum alpha feta protien - all testicular cancers raise AFP
Germ cell - raise afp and HCG
Gynacomasteia
Hyper thyroidsm

How well did you know this?
1
Not at all
2
3
4
5
Perfectly
111
Q

52 has so we pain down left side of the abdomen - worst pain whats the best investigation ?

A

Non contrast CT of the kidneys ureter and bladder - kidney stones but if pregnant it was US

How well did you know this?
1
Not at all
2
3
4
5
Perfectly
112
Q

What is an example of land field group A strep?

A

Streptococcus pyogenes - clindamycin

How well did you know this?
1
Not at all
2
3
4
5
Perfectly
113
Q

A 28 year old man attends A and E with a 3 month of history of night sweats , weight loss and a persistent productive cough with red blood what would you do ?

A

Ziehl Neelson stain on sputum - 1 pink bacilli is positive

How well did you know this?
1
Not at all
2
3
4
5
Perfectly
114
Q

Man is diagnosed with TB and starts treatment but is worried about red orange urine?

A

Rifamapicin

Isoniazid: Numb and tingly extremeties
Ethambutamol : Ocular side effects
Rifampicin is orange red urine
Pyrazinamide - arthalgia

How well did you know this?
1
Not at all
2
3
4
5
Perfectly
115
Q

What is coagulase positive

A

S.aureus

How well did you know this?
1
Not at all
2
3
4
5
Perfectly
116
Q

Which class of antibiotics target cell wall?

A

Glycopeptides, cephalosporins, penicillins and carbapenams

Macrolides inhibit protien synthesis and are used in cases of penicillin allergy

How well did you know this?
1
Not at all
2
3
4
5
Perfectly
117
Q

Which two species of malaria can persistent

A

P.ovale and P.Vivax - 14 day primaquine

How well did you know this?
1
Not at all
2
3
4
5
Perfectly
118
Q

A man presents for a routine visit with elevated ALT and a history of IVDY want to investigate for HBV what is indicative of active HBC

A

Hepatitis B surface antigen - HBsAG

How well did you know this?
1
Not at all
2
3
4
5
Perfectly
119
Q

75 year old man has a history of productive cough, dyspnoea and pleuritic chest pain with a low grade fever whats the most common cause of CAP

A

S. Pneumoniae ( atypical are mycoplasma , chalmydopjila , legionella - went on holiday stayed in hotel hot country )

How well did you know this?
1
Not at all
2
3
4
5
Perfectly
120
Q

What do you do first bloods or antibiotics in Sepsis?

A

Bloods

How well did you know this?
1
Not at all
2
3
4
5
Perfectly
121
Q

19 year old student has a fever non blanching rash and photo sensitive - lumbar puncture is turbid high opening pressure high WBC and low glucose whats happening

A

Bacterial mengitis

How well did you know this?
1
Not at all
2
3
4
5
Perfectly
122
Q

What’s the best way to detect viruses

A

PCR

How well did you know this?
1
Not at all
2
3
4
5
Perfectly
123
Q

What cells do HIV infect

A

CD4

How well did you know this?
1
Not at all
2
3
4
5
Perfectly
124
Q

What do you give staph infections

A

Flucloxacillin

How well did you know this?
1
Not at all
2
3
4
5
Perfectly
125
Q

What should you watch out for in antibiotic questions

A

Penicillin allergy ( usually give a macrolides

How well did you know this?
1
Not at all
2
3
4
5
Perfectly
126
Q

Mark is a 63 year old man coming to the clinic with dark stool. He complains of vomiting and occasionally having diarrheal. He has a 30 pack year history and drinks 22 units of alcohol per week. He feels abdominal pain that has been getting worse pver the last few months. He has been on ranitidine for 2 years now. He’s lost 9kg in the last few months. He has also been diagnosed with UC in the past. His bloods show anaemia and thrombocytosis whats the diagnosis?

A

Gastric Adenocarcinoma- melena and weight loss, haemetemesis

Colon adenocarcinoma would have fresh red blood mixed with stool

SCC are only in the oesophagus - progressive diffculty eating then swallowing

UC: Fresh red blood and mucus but no weight loss or constipation, diarrhoea

Haemorrhoids - blood when wiping not mixed with stool

How well did you know this?
1
Not at all
2
3
4
5
Perfectly
127
Q

Patient complains of abdominal pain that has been lasting months. it is localized to the epigastrium. The pain is the worst when he eats. He has been omeprazole and ranitidine but it has not madae the illness go away. He is then given a “test” that shows the patient has H.pylori. What is the necessary treatment and diagnostic test for this?

A

Omeprazole, amoxicillin and clarithrpmycin

Test: Stool antigen test or C-Urea Breath Test

How well did you know this?
1
Not at all
2
3
4
5
Perfectly
128
Q

John is an avid smoker with a 20 pack history. He has a family history of colon cancer. He has had recent weight loss. He explains that he has difficulty swallowing everything and so he has barely been able to eat, Many times when he tries to eat stuff, they tend to regurgitate. He also explains that he has shortness of breath at exertion and sometimes his fingers tend to get very white and cold. What is the most likely diagnosis?

A

Systemic sclerosis

Differential: Oesophageal Squamous cell Carcinoma but no progression.

Fibrosis all over the body. In lungs causes restrictive dyspnoea. In esophagus it stiffens causing no peristalsis. In small peripheral vessels causes raynaud’s.

How well did you know this?
1
Not at all
2
3
4
5
Perfectly
129
Q

Patient comes in with a 3-week history of severe constipation and abdominal pain. At the A+E, he looks very unwell and has vomited a few times since arriving at the hospital. He has T 38.2, HR 111 and RR 24. What is the most likely diagnosis? What is the gold standard investigation for this?

A

Diverticulitis - almost always with constipation and fever
Differential: appendicitis because only causes pain on the right
Gold standard: CT (can’t colonoscopy because perforation)
Diverticular disease: colonoscopy

This is caused by constipation causing unwell patient and fever

weakness in intestinal wall - poo gets stuck

How well did you know this?
1
Not at all
2
3
4
5
Perfectly
130
Q

How does volvulus present?

A

Coffee bean sign

No fever would be present

How well did you know this?
1
Not at all
2
3
4
5
Perfectly
131
Q

19 year old patient previously diagnosed with borderline personality disorder presents to the hospital extremely unwell. His eyes are barely open and is extremely confused and disoriented. His sclera has yellowed on examination. His mom explains that there was an empty bottle of capsules next to him but is not sure what it was.
What is the most likely drug he has overdose on?
What treatment does he need to recieve?
One of the nurses suspect it is Wilson’s disease, the doctor examines the patient’s eye and finds no clinical feature of it. What is the doctor looking for?

A

Paracetamol
Jaundice
n-acetylcysteine (IV)
Charcoal only if it’s been within the hour

Confused disoriented

Kayser fleischer ring

How well did you know this?
1
Not at all
2
3
4
5
Perfectly
132
Q

Name two features of acute liver failure

A
nausea
vomiting
malaise
Jaundice - itching
ascites
pain on right side

bleeding
coagulopathy
confusion
encephalopathy

How well did you know this?
1
Not at all
2
3
4
5
Perfectly
133
Q

Name two features of chronic liver failure

A

Ascites, gastro-oesophageal varices, peripheral oedema, jaundice triad, clotting dysfunction, malnutrition (bile not produced and liver not storing/processing nutrients).

134
Q

40, pain in his abdomen pain is in epigastrium radiating to the back, vimited, gallstones what investigation?

A

Serum amylase or lipase - acute pancreatitis
Hypercalceamia can cause it
alcohol and gallstones

135
Q

Where do you get gaurding?

A

Perforated appendicts -

136
Q

Jane contrinues drinkinh ahbits - pancreatic calcification?

A

Chronic pancreatitis, give

creaons and ceolaic is a risk factor caused by gallstones and alcohol, risk

137
Q

Max ia a 50 year old gentleman presenting to the A&E with acute pain in his right upper quadrant. This happened after she had a mukbang session eating Fish & Chips. He denies any chest pain, SOB or recent trauma. He had felt nauseous but has not vomited. He bowel movements are normal. He had a past medical history of gallstones. Max leads a sedentary lifestyle and stays at home 100% of the time. His obs is only remarkable for raised temperature. O/E RUQ tenderness, positive murphy’s sign
What’s the definitive treatment of this disease?

A

Laparoscopic cholecystectomy

Jaundice + cholangitis
Fever
Worse on fatty foods

RF for gallstone -> Fat, female, forty, fair

138
Q

Tom is a 55 year old gentleman with a history of sever alcoholic cirrhosis, he is confused and unable to giev a full history. He later comlains dark colouring of his stools. He has a past medical history of liver disease and hypternsion. His obs are temp 36.6, RR 18, PR 113, BP 80/60, mmHg & O2 98%
O/E confused hepatomegaly and jaundice
His relevant bloods are RBC 78 (90-110), WBC 80 (70-90), Urea 90 (70-80), Creatinine 75 (70-80)
Diagnosis?

A

Rapture of oesophageal varices

Blood loss
Hyptertension
shock
Urea -> internal bleeding

Mallory-Weiss tear -> self limiting
Peptic ulcer disease -> no epigastric pain and history of NSAIDs
Anal fissure -> bright red blood and straining down there

139
Q

Max is a healthy 23 yo male who had recently returned from sri lanka on a volunteering trip. He had not had any vaccinations or prophylaxis prior to the trip. HE reported symptoms of fever, nausea and vomiting, and also felt generaly unwell. Blood tests show IgM antibodies in response to the hepatisis A virus. O/E RUQ pain with hepatomoegaly
Cause?

A

Hepatisis A acquire by eating contaminated food

Hep A -> dirty food
Hep B -> unprotected sex
Hep C -> IVDU

higher ALT + higher billirubin
Dark urine & normal stools

140
Q

Julian comes to the GP complaining of yellowing to his skin. He freuquently gets pain in his RUQ after meals. He has had gallstones in the past and he thinks it might be the same problem again.
What wou;d be raised on his LFT?
What other symptoms would he have?

A

GGT and ALP bilirubin

Dark urine and pale stools

Gillbert causes pre-hipatic jaundice

141
Q

Name 4 risk factors for gallstone formation

A

dehydration, high cholesterol diet, obesity,

Fat, female, forty, fair

142
Q

Name 3 types of gallstones

A

Cholesterol, black/brown pigmented stones

143
Q

Black gallstone causes

A

sickle cell, systic fibrosis,

144
Q

25 yo man, GP with 3-month history of diarrhea, abdominal pain and weight loss. He undergoes a biopsy which confirms Cronh’s disease

List 3 diagnoses
Name a blood test an an imaging test to aid with diagnosis
List 2 histological features from biopsy
List 3 differences between uC and Cronh’s
Name a lifestyle modification for him
How to maintain remission for his condition?

A

Malignancy, uc, ibs, coeliac

diverticular

colonoscopy, cobblestone apperance, rosthrone ulcer, skip leisions, blood calprotectin

transmural inflamation

goblet cell

non-caseating

lympocytic infiltration

UC endoscopy
mouth ulcers
clubbing
apthous
pyoderna gangrenosum
eryhtmous nodusum
uveitis
episcleritis
anchilosing spondelitis
prednicalone acute
long line azathiprine
second line methotraxate

stop smoking

145
Q

A 21-yo female vomits blood after drinking

A

Mallory weiss tear

146
Q

38 yo man complains of intermittent abdominal pain. It’s relieved by eating. C-urea breath test is positive. Diagnosis? Appripriate drug treatment?

A

Duodenal Ulcer

Omeprazoe, clarithromycin and amoxicillin

147
Q

17 yo abdominal pain, bloating for 6 months, intermittent loose and smelly pale stools with no blood or mucus. Past history of B12 deficiency and mum is on levothyroxine for thyroid disease. She is pale , her abdomen is tender but soft with no guarding. Digital Rectal examination is normal. Which most likely appropriate investigation?

A

Coeliac of course

Total immunoglobulin A + IgA tissue translutaminase - 1st line serology

148
Q

80 to man 1 week ibality to open bowels or pass wind, denies previous episodes and presented due to abdominal distension, discomfort and nausea. Reports that noticed PR beeding several months ago but was painless and infrequent. Denies issues with constipation but notes that over the last 2 months he noticed more frequent loose stools and significant weight loss. An abdominal radiograph revelas dilated loops of large bowel. What is the likely cause of the obstruction?

A

Rectal cancer

149
Q

Which coronary artery is most commonly occluded in an MI

A

LAD V1-V4

150
Q

A 32 has lethargy and blurred vision and suspects MS which myelin producing cells are destructed:

A

Oligodendrocytes not Schwann cells !

151
Q

Which hormone acts on the leydig cells to produce testosterone

A

Lutenising hormones
FSH acts on Sertoli cells

GNRH causes release of LH and FSH

152
Q

24 2 month history, 3-4 loose motions with blood - UC is suspected which part of the bowel is affected?

A

Rectum - starts in rectum then spreads

Crohn’s is terminal ileum

153
Q

What sort of hypersensitivity is anaphylaxis?

A

T1 is within 1 hour IgE mediated and is anaphylaxis
2 is IgG or IgM hours to days e.g. haemolytic anaemia
3 is immune complex mediated 1-3 weeks and causes serum sickness and SLE
4 is T cell mediated days to weeks e.g. SJS and rash

154
Q

56 SCLC male is lethargic and disoriented - low sodium what is the mechanism of action of the hormone being secreted to excess?

A

Insertion of aquaporin channels- ADH

155
Q

Unilateral tremor, ridgitiy and bradykin ease what is dysfunctioning?

A

Substantia nigra

156
Q

28 year old female is referred to colposcopy - what time of epithelium lines the endocervix

A

Endocervix - mucous secreting simple columnar epithelium

Ectocervix? Stratified squamous non kertainxed

157
Q

How does testicular torsion present?

A

Acute onset testicular pain with absent cremasteric reflex

158
Q

A 65 year old male presents to outpatient clinic for his abdominal aortic aneurysm screening. He is found to have a dilation of the abdominal aorta just above the level of the bifurcation of the abdominal aorta into the common lilac arteries. The level of the bifurcation corresponds to which spinal level?

A

L4

159
Q

CT scan left precentral gyrus what defect what symptoms is it likely to result in?

A

Weakness of the right side of the body

160
Q

52 year old has a 5 day history of facial pain nasal congestion and headaches worse leading foreward - which sinus is most affected in sinusitis

A

Maxillary

161
Q

What does beclomethasone do?

A

Decreases cytokines formation , decreases micro vascular permeability ,reduces bronchiol hyperesponsivness and inhabits eosinophil influx

Smooth muscle is relaxed by salbutomal

162
Q

2 week wait colonoscopy - mass in sigmoid colon , mass in sigmoid colon biopsy confirms adenocarcinoma in the sigmoid which lymph nodes has it gone to ?

A

Inferior mesenteric nodes

Para aortic - testes ovaries and adrenal glands ?

163
Q

What is an effect of angiotensin 2?

A

Release of aldosterone

164
Q

Sudden thunderclap headache + Menigism + PKD history what is it?

A

SAH

165
Q

Cushings triad

A

Physiological nervous system : WIdening pulse pressure, bradycardia, irregular respiration’s

166
Q

62 year old female presents to ED with Abdo pain localised to RUQ jaundice what is it most likely to be?

A

Ascending cholangitis - charcots : Jaundice, RUQ and fever

167
Q

6 month history of weight loss etc etc - CML whats the abnormality?

A

T(9:22)

168
Q

Teenage boy has an injury from rugby - mid shaft humeral fracture whats at risk?

A

Radial nerve -

Neck - axillary nerve
Mid shaft - radial
Lower supachorniol - medial nerve and brachial artery in front of

169
Q

What is the mechanism of action of ondansetron?

A

5-HT3 receptor Antagonsit

Beta blocker CVD
H1 - antiemetic on histamine - ethinze
D2 - dopamine - domperiomde ad methocloparime
NK-1 neurokinine -

170
Q

What does the right testicular vein drain into

A

IVC

171
Q

Process of columnar epithelium into mature sqaumous epithelium is what

A

Metaplasia

172
Q

38 year old female has a 6 month history of fatigue blood tests below what’s that anaemia cause? Low MCV

A

Iron deficiency

173
Q

What is the ovarian cancer bio marker?

A

CA 125

CA 19-9 - pancreatic
CEA- colon
PSA - prostate
HER- 2 breast cancers and BRACA

174
Q

What do penicillins do?

A

Inhibit cross linking in peptidoglycan cell walls?

175
Q

What epithelium lines the oesophagus?

A

Stratified sqaumous non keratinised

176
Q

A 47 year old male has an AKI = potassium 6.5 which ECG changes are hyperkalemia

A

Tall tented T wave

177
Q

What is a node of ranvier?

A

A junction between two Schwann cells

178
Q

A 71 year old has stiff hands worse in the mrinubg - periarticalr erosions and loss of joint space

A

Rheumatoid arthritis

179
Q

What do bronchiole walls lack

A

Hyaline cartilage -

Bronchi - bronchiole is

Primary, secondary, tertiary - lobar - segmental bronchi ( C shaped hyaline Cartilage ) smaller airways dont need it

180
Q

Obstructuvive v.s. Restrictive

A

Measure air you can expel
Obstructive: FEV1 low FVC normal ratio abnormal
Restrictive: FEV1 and FVC low normal ratio

181
Q

Which bacteria usually causes resp symptoms:

A

Haemophilus influenzae
Pseudomonas - skin and wounds
UTI -e.coli

182
Q

Which electrolyte do you need to watch in DKA?

A

Potassium - in trolled lypoliss- excess ketones

183
Q

72 tear old is hypotensive, tachycardia, abdo pain high lactate, AF, what next?

A

CT angiogram - thrombus SMA

184
Q

78 year old female stroke ward - right MCA unsafe swallow, pyrexial, tachypnoeic and tachycardia - aspiration pneumonia which lobe is affective?

A

Right middle lobe

185
Q

Patient and parter - husband has sickle cell and they have a son with it no PMH what is the probability of them having another child with sickle cell?

A

50

186
Q

72 memory clinic - dementia what screening test?

A

MOCA

187
Q

40 year old Caucasian - blood pressure over 150/100 whats first line

A

Ramipril

188
Q

22 female - lethargy ABdo pain, diarrheo and weight loss , IDA

A

Biopsies are taken duodenum and jejenum

189
Q

Where would you find ciliates columnar epithelium with goblet cells?

A

Trachea

190
Q

What kind of cartilage makes up the menisci?

A

Chondrocytes and fibroblasts surrounded by type 1 and type 2 collagen

191
Q

What are the main types of cartilage?

A

Hyaline - most widespread , avascular , skeleton of the fetus, resp tract up until bronchioles, chondrocytes

Elastic: Avascular surrounded by perichondium, 2 collage, more elastic tissue, external ear, epiglottis and auditory tube

Fibrocartilage - avascular no perichondium, T 1 collagen , intervertebral disks, mandible, sternocalvicualr and pubic sympshss , aligns along lines of stress

192
Q

What does a vein not contain?

A

Internal elastic lamina

Tunica intima, media and adventitia

193
Q

Which layer do atherosclerotic plaques form

A

Tunica intima

194
Q

Skin functions?

A

Protection, water barrier, body tempurature regulation, non specific defence, excretion of salt synthesis of vitamin D, sensory organ

Merkel s cells are neural cresct
Langerhands are dendritic c- A trap cells

195
Q

What are the agranulocytosis

A

Lymphocyte and monocytes

Neutrophils, eosphinols and basophils do

196
Q

Where do you find osteocytes

A

How’s ship lacunae

Haversian down and volkammans across

197
Q

Describe a hepatic sinusoid

A

Blood in, bile out
Triad on edges and porta; vein in the Middle
Stellate- stoors nad scars
PERIS stain - Kupffer cells blue due to hemosederin

198
Q

What are the symptoms of hypocalcaemia:

A

Convulsions, arrhythmias, tetany, numbness, chvosteks and trousseaus

199
Q

Myeloma symptoms

A

C: Hypercalcaemia : Groans ( constipation), stones, bones and moans ( psychic)
R: Renal injury ( AKI, creatinine, high urea, low eGFR)
Anaemia: Breathlessness, fatigue
Bone destruction: Pathological fractures

200
Q

Which type of antibody does warm AIHA involved:

A

IgG

201
Q

What can you see in G6PD?

A

Bite cells, Heinz bodies

202
Q

What is a decreased haptoglobin a feature of?

A

Haemolytic anaemia

203
Q

What do blood tests show in haemolytic anaemia:

A

Normocytic, increased LDH, increased uncojugated bilirubin, decreased haptoglobin, increased reticulocyte count

204
Q

What’s sickle cell pathology?

A

Beta globin chains on chromosome 11 misshapen , normally glutamic acid to valine and results in HBs - aggregates with other HBs and RBC polymerises when deoxygenated causing a sickle cell red blood cell
Fetal haemoglobin is 2 alpha and 2 gamma - no symptoms until a few motnhs

205
Q

Give signs of Polycythaemia?

A

High haemoglobin, facial flushing, intermittent itching, worse in the path, alcohol, high altitude, obstructive sleep apnoea, JAK2 mutation

206
Q

What are the two types of Polycythaemia?

A

Primary: JAK2 mutation - Polycythaemia Vera
Secondary: Alcohol, chronic hypoxia , obstructive sleep apnoea, living at altitude , lung disease, smoking, EPO secreting tumours,

207
Q

Sam has a 3 year old bloody diarrhoea for 5 days after eating some dodgy cottage pie at nursery , purpurin rash, fatigue not producing much urine, oliguria,

A

Haemolytic uraemia syndrome : Microangiopathic haemolytic anaemia, thrombocytopenia and renal impairment
DIarrheoa shiga toxin associated
FBC: Anaemia and thrombocytopenia, haemolysis , LDH and bilirubin, peripheral blood smear and schistocytes, U and E shows raised creatinine and electrolyte abnormalities , supportive

208
Q

Fever, chills , Diarrhoae , Africa, mosquitos , no prophylaxis, 38 degress and mild jaundice whats the treatment

A

Oral hydroxychloroquine

209
Q

How do you diagnose malaria:

A

Giemsa, thin and thick blood films
Mild - oral hydrozyquinie
Sevre is IV artesunate and priaquine - notify PHE h
SLE is also treated with anti malarials

210
Q

What does IDA show on film

A

Microcytic hypochromic RBC ( anisocytosis and poikilocytosis )

Also TAILS

Signs: Koilonychia, angular stomatitis, atrophic glossitis , pallor

211
Q

A 70-year-old male complains of pain in his upper abdomen, with fever, night sweats and weight loss. Examination reveals enlargement of his liver and spleen. A full blood count shows raised basophils, eosinophils and neutrophils. Which of the following would most likely be implicated in this scenario?

A

Philadelphia chromosome is in 95% of patients with chronic myeloid leukaemia ( CML ) translocation between the long arm of 9:22 resulting in an ABL proto-oncogene ese from chromasone 9 being fused with BCR gene from chromosome 22 - the resulting BCR-ABL codes for a protien with tyrosine kinase activity

50-70
anaemia an dlethargy 
weight loss and sweating
splenomegaly
granulocytes increase
212
Q

Q.2 A 24-year-old woman with known sickle cell disease presents with sudden onset severe right leg pain and 6 days of fever. She has noticed that her right leg is slightly swollen, and she is unable to weight bear on the affected leg. Which of the following would most likely cause her symptoms?

A

Osteomyelitis is usually S.Aureus but in sickle cell its salmonella

Diffrentials are capilalry infarction secondary to intravascular sickling

213
Q

Q.3 A 68-year-old male presents with exertional fatigue. He is otherwise well with no significant past medical history, and he is not on any medications. His blood results highlight the following abnormal readings:

A

Multiple myeloma:

Bence jones protiens cause RBC to aggregate - rouloux
C
R
A
B
lethargy, infection, hypercalcaemia, renal failure, carpal tunnel, neuropathy, hyperviscocity

Monocolonal invesyogations, bence jones, rain drop skull which is different from primary hyperpaprathyrodism pepper pot

214
Q

Where are smudge cells seen?

A

Chronic lymphocytic

Chronic lymphocytic leukaemia (CLL) is caused by a monoclonal proliferation of well-differentiated lymphocytes which are almost always B-cells (99%). It is the most common form of leukaemia seen in adults.

Features
often none: may be picked up by an incidental finding of lymphocytosis
constitutional: anorexia, weight loss
bleeding, infections
lymphadenopathy more marked than chronic myeloid leukaemia

Investigations
full blood count: lymphocytosis, anaemia
blood film: smudge cells (also known as smear cells)
immunophenotyping is the key investigation (confirms CLL)

215
Q

A 58-year-old man presents to his GP with increasing tiredness, accompanied by bruising on his legs. He also complains of aching bones. He has no previous illnesses. On examination, he is pyrexial and pale, has bony tenderness over the sternum and tibia, and has petechiae on his legs. A bone marrow biopsy was taken and stained, showing the following: Auer rods

A

AML

cute myeloid leukaemia is the more common form of acute leukaemia in adults. It may occur as a primary disease or following a secondary transformation of a myeloproliferative disorder.

Features are largely related to bone marrow failure:
anaemia: pallor, lethargy, weakness
neutropenia: whilst white cell counts may be very high, functioning neutrophil levels may be low leading to frequent infections etc
thrombocytopenia: bleeding
splenomegaly
bone pain

Acute promyelocytic leukaemia M3
associated with t(15;17)
fusion of PML and RAR-alpha genes
presents younger than other types of AML (average = 25 years old)
Auer rods (seen with myeloperoxidase stain)
DIC or thrombocytopenia often at presentation
good prognosis

216
Q

What does Rituximab do?

A

Binds to CD20

Rituximab is a monoclonal antibody that is used in various conditions including CLL and Non-Hodgkin’s Lymphoma. It acts by binding to the CD20 protein on the cell surface of B-cells and induces lysis. All the other answers are red herrings and are just there to confuse you.

217
Q

Q7: 68F has been admitted to the frailty unit after a fall at home. On examination she has a dislocated hip and lacerations over her body. She has a BMI of 30 and is on HRT for her post-menopausal symptoms. You consider her to have a significant risk for VTE and begin her on prophylactic treatment. What is the mechanism of action for the prescribed medication?

A

LMWH

Patient presents with signs of DVT and this is confirmed using a doppler ultrasound. Prophylactic treatment for DVT is using LMWHs and this is the MOA.
LMWH works by binding to ATIII and inhibiting factor Xa. (indirect inhibition)
Warfarin works by inhibiting the reduction of Vitamin K and thus inhibiting production of factors 10,9,7,2 (1972)
DOACs such as apixaban directly inhibit X 🡪 1st line treatment
This is how UH works.
This is the MoA of clopidogrel, an antiplatelet. You wouldn’t use this for DVT.

218
Q

Whats in Curb 65?

A
Confusion
Urea > 7 mmol/L
Respiration rate >30
BP <90 systolic OR <60 diastolic
65 - aged over 65 years

0-1 - Amoxicillin 500mg TDS (Doxycycline 200mg OD then 100mg OD, Clarithromycin 500mg BD)

1-2 - Amoxicillin 500mg TDS PLUS Clarithromycin 500mg BD

3+ - Coamoxiclav 500/125mg 3 TDS orally or 1.2 g TDS IV (levofloxacin) PLUS Clarithromycin 500mg BD

219
Q

How do you treate achalasia

A

CCB+nitrates you can do balloon diltation and BOTOX

220
Q

What are LFTs in alcohol abuse?

A

High AST, high ALT, high GGT

221
Q

Q8: A 5 year old female presents to A&E with a widespread purpuric rash and recurrent episodes of epistaxis. Parents report that she has been recently ill with Covid-19 two weeks ago. She complains of headaches and nausea while seated in the A&E department. Bloods show a markedly low platelet count. There is no FHx of bleeding disorders.

A

Immune thrombocytopenic purpura

ITP is most common in ages 2-6 years old and presents with purpura and nosebleeds. It can also cause N+V, headaches, and bleeding gums. It is an autoimmune condition that can present after a viral infection and is self-limiting in nature. It is treated with prednisolone, IVIg and platelet transfusions.
TTP presents similarly, however has a different pathophysiology and is more common in adults. TTP is a lack of the VWF cleaving protein and on blood film will show fragmented erythrocytes as well as raised creatinine, bilirubin and reticulocytes.
SJS and bullous pemphigoid are dermatological conditions and do not present this way but are just there to confuse you.
Haemophilia doesn’t usually present this way and has a massive genetic component as well as being a largely male disease.

222
Q

A 3Y old male patient with Down’s Syndrome presents to clinic with a two week history of shortness of breath and fatigue. On examination you find hepatosplenomegaly, lymphadenopathy and drooping of the right side of his face.

A

Acute lymphoblastic leukaemia

ALL is more common in children aged 2-4 years and has an association with Down’s syndrome. ALL affects the lymphoblasts and therefore presents with symptoms of anaemia, thrombocytopenia and neutropenia.
AML presents similarly but is much more common in adults and therefore is unlikely to be the answer in this situation.
CML also affects adults and presents with B-symptoms such as weight loss, fatigue, night sweats and bone pain.
Hodgkin’s lymphoma has a bi-modal incidence and affects young adults and the elderly and characteristically in exam questions has the patient experiencing pain when drinking alcohol.
Kawasaki’s disease is a vasculitis and doesn’t present this way.

223
Q

What do you know about hodgkins?

A

Hodgkin’s Lymphoma classically presents with the presence of Reed-Sternberg cells. It is also seen with B-symptoms and non-tender rubbery lymph nodes.
ABVD is the mnemonic to remember for the chemotherapy drugs that are used to treat Hodgkin’s.
A- Adriamycin
B- Bleomycin
V- Vinblastine
D- Dacarbazine

224
Q

A 31 year old obese patient came into A and E complaining of a painful, swollen leg. She has a past medical history of breast cancer (undergoing treatment), antiphospholipid syndrome, and varicose veins. Her current medications include her chemotherapy regimen, vitamin D and an IUD (intrauterine device). The results of her investigations are as follows: D dimer test (positive), Proximal leg vein ultrasound (Confirms DVT). Which of the following is not a risk factor for her DVT:

A

Contraceptive device

Risk factors for DVT:
Obesity 
Recent surgery 
Long haul flights 
Active cancer/chemotherapy 
Immobilization 
Dehydration 
Oral contraceptive 
Hypercoagulability (i.e. Antiphospholipid syndrome)
Previous DVT/PE
Varicose veins
Pregnancy 
Major trauma 

D dimer is sensiitve ( true negatives ) not specific ( true positives )

DOACS
LMWEH

225
Q

A 62-year-old man has poorly controlled type 2 diabetes. His GP started him on gliclazide treatment in addition to metformin with the aim of improving his glycaemic control. However he presents to the local emergency department 3 days later after feeling tired and very SOB. When asked about any allergies, he remembered when a doctor told him to not take primaquine due to an inherited disease - but he doesn’t remember its name. On examination he appears visibly jaundiced. A blood film reveals the presence of Heinz bodies and bite cells.
What investigation would confirm the diagnosis?

A

G6PD enzyme assay

Glucose 6 phosphate dehydrogenase deficiency is a X linked inherited condition which predisposes a patient to haemolytic anaemia due to oxidative stress. The crises in this case was aggravated by gliclazide

Jaundice, pallor, SOB, anaemia, dark urine, nausea, splenomegaly

226
Q

A 44 year old man comes into A and E with significant bruising, epistaxis, fever and gingival bleeding. He ends up being diagnosed with DIC. FBC analysis reveals low fibrinogen levels, what blood product can be used to treat this?

A

Cryoprecipitate

PCC usually given for the reversal of warfarin during bleeds, head trauma or prior to surgery
FFP can be used to treat clotting abnormalities (PT or APTT elevated)
Packed red cells for chronic anaemia
Beriplex is a type of PCC. (Note vitamin K can also be given to reverse warfarin)
Cryoprecipitate is the correct answer. It is rich in fibrinogen

227
Q

70 Y/O male patient with an advanced, high-grade liver cancer is undergoing an intense chemotherapy regimen. The patient has a past history of CKD and alcohol dependence.
Which of the following drugs should be prescribed to the patient during their cancer treatment?

A

Allopurinol - xanthine oxidase inhibtors

228
Q

What is used in acute alcohol withdrawel?

A

Chlordiazepoxide is a benzodiazepine used in acute management of alcohol withdrawal. Not suitable here as even though we are told the patient has a PMH of alcohol dependence, we are not told he is currently experiencing alcohol withdrawal

229
Q

A 87 year old patient comes into clinic complaining of lethargy, SOB, weight loss, and left upper quadrant discomfort. On examination, sub conjunctival pallor and splenomegaly are noted. The patient has a number of investigations and is, subsequently, diagnosed with philadelphia chromosome chronic myeloid leukemia. What is the first line management?

A

Imatinib - tyrosine kinase inhibtor

hydroxyurea can be used but is not first line

230
Q

A 33 y/o lady presents with headache, fever, and diarrhoea. She mentions she recently returned from a month-long trip to Djibouti. You order a thick and thin blood film and subsequently diagnose her with malaria. You prescribe her chloroquine as well as primaquine.
Which species of malaria require treatment with primaquine (as well as the normal treatment)?

A

plasmodium vivax and ovale

231
Q

A 17 year old presents to your gp clinic with fatigue, pallor and dyspnea upon exertion.Upon further history, you find that she is a vegetarian and has had heavy periods . Her vital signs are all normal.

a ) Name 3 causes of microcytic anaemia
[2]
b) Name 3 causes of macrocytic anaemia

A

Microcytic : Thalassemia, IDA, lead deficiency, sideroblastic

Macrocytic: B12 and folate deficiency, alcohol use, reticulocytotics

232
Q

give some signs and symptoyms of anaemia

A
Pallor
Beefy red tongue ( b12 deficiency)
angular cheilitis(b12)
Hyperdynamic circulation if sevre
Jaundice (haemolytic anaemia)
Fatigue
Dyspnoea
Faintness
Palpitations
Headaches
Tinnitus
233
Q

What differentiates megoblastic and normoblastic on film?

A

Hypersegmented neutrophils and megoblasts

234
Q

A 5 year-old boy from India presents to your GP practice, his mother complains that he is constantly tired, failing to grow and has gradually been developing a large tummy. On examination, you note chipmunk facies, pallor, hepatosplenomegaly, and jaundice. The mother also states that they have a family history of dysfunctional RBC

A

Thalassemia

Alpha is far east beta is mediterranian, asia, africa

FBC: microcytic anaemoa, WBC, platlets 
Peripheral smear 0 microcytic hypochromic
Skull X ray : hair on end
Genetic testing 
Gel elctrophoresis is gold 

Minor treat: non
Intermediate: not transfusion dependent
Major: transfusion dependent and chelation to prevent iron overload

235
Q

A 77 y/o male patient presents to A&E with with severe rectal bleeding and vomiting blood. You notice on the drug charts that the patient usually takes warfarin due to recurrent clots in the past. The patient mentions that he recently got back from a 7-day bender in Ibiza and had forgot to take his warfarin during that time. When he got back, he decided to take a weeks worth of tablets to make up for the ones he missed.
What would you prescribe to this patient to reverse the effects of warfarin? (Give 2 examples)

A

Vitamin K or Beriplex

Warfarin inhibits hepatic production of vitamin K-dependent coagulation factors and cofactors. Vitamin K must be in its reduced form for synthesis of coagulation factors. It is then oxidised during the synthetic process. An enzyme called vitamin K epoxide reductase reactivates oxidised vitamin K. Warfarin inhibits vitamin K epoxide reductase which prevents reactivation of vitamin K and coagulation factor synthesis.

236
Q

A 58 y/o presents to clinic with fatigue, pallor, and headache. He mentions that his symptoms began a few weeks after undergoing an ileocecal resection to remove a tumour. A blood count and film is carried out and the patient is subsequently diagnosed with pernicious anaemia.
Name 5 other signs and/or symptoms of pernicious anaemia:

A
Parasthesia
Pallor
Fatigue 
Headache
Numbness
Peripheral neuropathy
Pallor
dyspnoea. anorexia, tachycardia, lemon yellow skin , red sore tongue ( glossitis ) angular stomatitis, loss of proprioception, weakness and ataxia
237
Q

A 26 year old male presents with haemoptysis, a strong cough, wheeze, chronic nosebleeds and a saddle-nose deformity. A urine dipstick reveals haematuria and proteinuria. Blood test reveals cANCA positive.

A

Granulomatosis with polyangitis

cANCA positive
Hearing loss, sinusitis, nose bleeds
Saddle shaped nose due to perforated septum
Treated with steroids and immunosuppressants

238
Q
  1. Mary is a 62y/o lady presenting with dry eyes. She has a PMH of SLE, for which she takes hydroxychloroquine. Mary tells you she has also been more tired than normal lately, and when you ask about her diet she tells you she drinks a lot of water as her mouth gets very dry.
A

Sjorgens syndrome
Secondary as she already has SLE
Bloods - anti ro and anti la , ANA,RF , CRP, ESR, FBC
Schrimers eye test and the and rose benagl staining
Lymphocyte mediated autoimmune disease characterised by destruction of the minor glands, lacrimal glands and joints
Symptomatic ce..g eye drops, vaginal lubricants, muscarinic agonists
triad of Xerostomia, Xerophthalmia and inflammatory arthritis

can lead to NHL an itchiness and dryness in vagina, dysphagia, gastritis MALToma, Pericarditis, CVS

239
Q

What are the complications of SLE?

A
Key features:
Photosensitive malar rash - butterfly rash that gets worse with sunlight
Arthralgia
Myalgia 
85% positive for ANA
70% anti-dsDNA positive
Cardiovascular disease
Infection 
Anaemia of chronic disease
Pericarditis
Pleuritis
Interstitial lung disease
Lupus nephritis 
Neuropsychiatric SLE 
Reccurent miscarriage 
Venous thromboembolism
240
Q

An 18 year old male presents to his GP with progressive back pain and stiffness. You decide to send him for a spinal x-ray to investigate which reveals a bamboo spine on the x-ray.

What gene would you associate with this disease?
Name 3 other findings you would expect to see on the x-ray.
Give an example of an Anti-TNF drug that you would prescribe this patient.

A

Ankylosing spondylitis, HLA b27

Squaring of the vertebral bodies
Subchondral sclerosis and erosions,
Syndesmophytes
Ossification of the ligaments, discs and joints,
Fusoion of the facet , sacroiliac and costovertebral joints

Etancerpt, adalimubab

241
Q
  1. Margaret is a 61-year-old dear who has limped into your GP surgery looking in discomfort. She says she has pain in her left knee, which is worse when she has to walk down to the shops or walk her dog. When you ask about joint stiffness, she says they are stiff for about 25 minutes when she wakes up, and then after that it becomes the pain that limits her. It has gone on for some years but she has had enough.

What is your differential diagnosis?
What might you see on an X-Ray of her knee?
What is the cause of these changes? (think about
cartilage physiology)
Other than analgesia, what management options does Margaret have?

A

1, Osteoarthritis
2. Loss of joint space, osteophytes, subchondral sclerosis and subchondral cysts

  1. Loss of bone cartilage , weaker - bone on bone surface erosion ) imbalance between cartilage being worn down and chondrocytes repairing it - structural joint issues
  2. NSAIDS, sodium hyaluronate injections
    Intraarticular steroid infection and joint replacement
242
Q

What are S+S of osteoarthritis?

A

Assymetrical monoarticular arthirits, joint pain, decreased function, stiffness less then 30 minutes, increased with activity

243
Q

What are risk factors of osteoarthritis?

A

Age, Sex, obesity, joint injurie, stress, genetics, bone deformities, diabetes, hemochromatosis

244
Q
  1. A 66 year old patient presents to the GP with bone pain. They have recently had a few fractures which resulted in them going to A+E. You also notice that when they walk in there is some bone deformity in their left leg. You send them for some testing.

How would the skull in this patient appear on x-ray?
Which LFT would you expect to be raised?
What type of drug would you use to treat this patient?

A

Pagets disease of the bone

Cotton wool appearnce due to patches of osteoblast and osteoclast activity- patches of increased and decreased density

ALP

Bisphosphonates

245
Q

What hand things do you see in RA v.s. Osteoarthirtis?

A

Osteoarthritis: Boutinerres, swan neck, ulnar deviation and z thumb, nodules

Bouchards and heberdens

246
Q

What are some extra-articular manifestations of RA?

A

Episcleritis, Pericarditis, raynauds, peripheral neuritis, anaemia, raynauds

247
Q

How do you manage RA?

A

DMARDS: Methotrexate, leflunomide, sulfasalzaine

2 of the above

3: Add in a biologic e..g TNF alpha infliximab, adalinumab, etancaerot
4: Methotrecate and rituximab

248
Q

What are some RA complications?

A

Feltys syndrome: RA + Splenomegaly+ neutropenia

Anaemia of chronic disease
Pulmonary fibrosis
Amyloidosis

249
Q

What are some diffrential diagnosis of joint pain and stiffness?

A
Osteoarthritis - degenerative arthritis
Polymyalgia Rheumatica: Inflmmatoru disorder causing pain, stiffness inflammation in the shoulders, neck muscle nad hips 
Gout 
Septic 
RA
250
Q
  1. Taylor presents to their GP complaining of muscle pain, fatigue and weakness in both hands and forearms, and their shoulders over the last few weeks. On examination you find Gottron lesions on their knuckles and elbows, a photosensitive rash on their face and some subcutaneous calcinosis. You conduct a blood test and their results show positive for ANA, a creatinine kinase of 1200 U/L, and Anti-Jo-1 antibodies.

What is the differential for this patient?

A

Dermatomyositis

Dermatomyositis = chronic inflammation of the skin and muscles

Polymyositis = chronic inflammation of the muscles (only)

Creatinine Kinase: normally under 300
Also raised:
Rhabdomyolysis 
AKI
MI
Statins
Strenuous exercise
Muscle pain, fatigue, weakness
Occurs bilaterally
Typically affects proximal muscles
Shoulder and pelvic girdle
Develops over weeks

If there is no skin involvement = polymyositis

If there is skin involvement = dermatomyositis

251
Q

What are some skin symptoms and antibodiees in dermatomyositis?

A
Skin symptoms:
Gottron lesions (erythematous plaques)
Photosensitive erythematous rash on back, shoulders and neck
Purple rash on face and eyelids
Periorbital oedema 
Subcutaneous calcinosis

Antibodies:
Anti-Jo-1: poly+derm
Anti-Mi-2: derm
ANA: derm

252
Q

9a. Vitamin D metabolism is important for the absorption of calcium and phosphate from the intestine, therefore important for bone mineralisation.

Which of the following substances is not involved in bone metabolism?

Parathyroid Hormone
1-alpha-hydroxylase
Antiphospholipid antibody
Oestrogen
Calcitonin
A

Antiphospholipid antibody - thrombosis and miscarriages

Oestrogen regulates bone metabolsism and inhibits remodelling protective against osteoporosis!

Calcitonin lowers blood calcium

253
Q

9b. Vitamin D metabolism is important for the absorption of calcium and phosphate from the intestine, therefore important for bone mineralisation.

i. What condition might cause an adult to have brittle bones due to insufficient vitamin D?
ii. What condition occurs in a child?
iii. Why do you not get this condition in adulthood?

9c. i. What does a T-Score of -2 mean?
ii. What is the first-line treatment for osteoporosis?

A

Osteomalacia

Rickets

Epiphyseal plates fused already

Osteopenia
Bisphosphonates

Osteomalacia & Ricketts

Osteomalacia is a disease of defective bone mineralisation, due to Vitamin D deficiency.

Presentation: fatigue, bone pain, bone and muscle weakness, and pathological / abnormal fractures.

Investigation: serum 25-hydroxyvitamin D low, serum calcium & phosphate low, serum ALP high, PTH high, z-ray may show osteopenia, DEXA may show low bone mineral density

Management: vitamin D supplementation (colecalciferol)

254
Q

Give three bisphosphonates?

A

Alendronate, risendronate and zolendronic acid

255
Q

How does the T score work?

A

More then -1 is normal
-1 to 2.5 is osteopenia
-2 to -2.5 osteoporosis
Add a fracture and its sevre

256
Q
  1. Susan is a 55 year old female who has presented to you with a severe unilateral headache in her forehead. She has also had some blurred vision and tenderness of the scalp. Her blood test results in an ESR of 73.
A

Do a temporal artery biopsy if it shows multinucleated giant cells - GCA

Raised ESR
Give 40-60mg pred

257
Q
  1. Which of the following is not a true regarding gout?

Negatively birefringent on polarised light microscopy
Monosodium urate crystals
Allopurinol is first-line treatment during acute flare
Tophi may deposit within the ear
Needle-shaped crystals

A

Allopurinol is not first line in an acute flare ! - colchine + NSAIDS are is

258
Q

Where does pseudogout usually present?

A

Knee - chondrocalcinosis is pathognmonic

259
Q

Sharon is a 54 year old female who presents to your GP practice with a main complaint of chronic widespread pain and tenderness. She complains that her muscles feel stiff in the morning. She also has had difficulty sleeping, and therefore is tired and has low mood and concentration. You determine that the pain is musculoskeletal, not arthritic.

What is your diagnosis?

A

Fibromyalgia - risk factors are psychosocial

Fibromyalgia = Disorder characterized by widespread musculoskeletal pain accompanied by fatigue, sleep, memory and mood issues

Poor diet can increase fibromyalgia pain, but isn’t a risk factor.

Repeated nerve stimulation results in the sensitization of the brain’s pain receptors, causing them to overreact to stimuli

Severe pain in 3-6 different areas, or milder pain in ≥7

260
Q

What are some risk factors for fibromyalgia

A
Neurosis: depression, anxiety, stress
Dissatisfaction at work
Overprotective family or lack of support
Middle age
Low income
Divorced
Low educational status
261
Q
  1. Colin is a 78-year-old man who has a long history of osteoarthritis. He underwent a knee replacement 3 days ago, and now has an acutely swollen knee that is hot to the touch, stiff, and he can’t move it due to severe pain. He has a fever of 38.1 and is lethargic.
A

Septic arthirits- S.aureus
joint aspiration and culture

Flucloxacillin + rifampcin

262
Q

Jules is a 14 year old who has presented to you with a complaint of bone pain with swelling around their knee. The swelling hasn’t gone down despite ibuprofen use over the last week. Jules has a PMHx of hereditary retinoblastoma.
Based on their symptoms and PMHx:

A

Osteosarcoma

X ray. MRI , biopsy

Surgery

263
Q

Mary is a 37 y.o lady presenting to her GP with a 2-week history of difficulties swallowing. She has no problem drinking but solid food tends to get stucked somewhere around her sternum. There is no pain on swallowing and she has not regurgitated food back out.
She denies any weight loss, fever or recent trauma.
Past medical history of GORD. Her mum had a disease which she could not recall but affected her skin badly.
O/E: Patches of shiny skin bilaterally on her arms up to her shoulder. Limited ROM of fingers. White discoloration over her knuckles
Based on the most likely diagnosis, which would be most useful for diagnosis of the pathology? (+++)

A

Serum nucleur antigens -

Limited - anticentromere

Diffuse - Antitopoisomeras, Anti scl70 and anti RNA polymerase 3

264
Q

Chimdi is a 1 year-old boy who had recently moved from Nigeria to the United Kingdom. During his 1st week here, he started having fever and felt generally unwell. When you examined him, he was tender to touch over his left hip. His vitals are temp: 38, RR: 40, 120bpm, 95 O2%. Later, you had found out from his parents that he had sickle cell disease.
Further investigations were ordered and it was confirmed that Chimdi had osteomyelitis of his left hip and was given antibiotics for it.
Based on the history above, what is the most likely causative organism? (++)

A

Salmonella

265
Q

Malcolm is a 25 y.o gentleman presenting to his GP with pain in his lower back for 3 months. He also gets stiffness in the back in the morning, however that improves with exercise. He has a family history of arthritis. He has Crohn’s for which is well controlled with medications.
His obs are normal. On examination, he has redness in both eyes. Based on the above, what is the most likely diagnosis? (+)

A

Ankylosing spondylitis

Psoriatic : psoriasis, nail pitting and sausage
RA: females
Reactive: Cant see, cant pee cant climb, recent infection
Osteoarthritis : Worse with ifnection

266
Q

What are some key signs of seronegative spondyloarthropathy?

A
Sausage digits
Psoriasis
Inflammatory back pain
NSAIDS
Enthesitis ( heel )
Arthritis
crohns and colitis / elevated CRP but can be normal 
HLA - B27
Uveitis
267
Q
A 82 y.o gentleman presents to the GP with a 2 month history of back pain. The pain is localised and does not radiate at all. He has also been feeling very tired recently and you notice that he seem to look really pale. Over the past 2 months, Jim been constantly getting pneumonia. Jim used to work in the petrol industry.
His obs are normal.
His relevant blood results are:
Hb: 97 (110-130)
Creatinine: 260 (<176)
Urea: 49 (<30)
CRP 63 (<40)
Based on the history, which is the best investigation to confirm the diagnosis. (++)
A

Multiple myeloma: CRAB and serum electrophoresis ( IGG A or light protien chains _

Clonal bone marrow greater than 10% + CRAB + serum electrophoresis, prognostic is CRP an dLDH

268
Q

Malcolm has a red hot swollen joint. He also has a fever and is systemically unwell. His blood pressure is low.
How would you manage this patient? (2)

Name 3 risk factors for septic arthritis? (1)

Malcolm is sexually active, what is the most likely causative organism for his septic joint? (1)

A

Sepsis 6 and joint fluid aspiration

Immunosuppresents diabtes , HIV old age, IVDU

Neisseria gonorrhoea

269
Q

63 y.o. Patient comes in with left sided knee pain. She says she’s had it for a few months now and hurts whenever she tries to move it. She also thinks she’s been having hearing difficulties on her right ear as of late. She is given a bone isotope test thats shows areas of abnormal bone turnover with decreased osteoclast action. The main areas found affected were the right side of skull and left knee.
What is the most likely diagnosis:
What is the first line treatment for her:

A

Pagets

Zolendronate

270
Q

John 42 y.o. Man comes to clinic with freaking out as he is afraid that he has lung cancer. He screams that he has recently had a cough and has been coughing out blood. He says he has hd weight loss recently too. His father had lung cancer while his mother had an autoimmune disease that he can’t remember the name of. He says recently he has been peeing less and the color is usually very dark reddish brown but he just thought it was because he had been drinking too much alcohol.
When investigated, his blood showed that he was positive for Anti glomerular basement membrane antibodies.
Urine dipstick showed:
Blood +++
Protein +
Nitrites
What is the most likely cause of his symptoms:

A

GOODPASTURES - affects lungs and glomerulus - anti GBM

271
Q

Mary 33 y.o. Woman comes into clinic complaining of pain and swelling of joints all over. Her fingers, wrist and knees hurt the most on both sides. She says that the pain and stiffness is worse in the morning but thankfully gets better as the day goes by.
What is the best biochemical blood marker to test for to diagnose her:
Name two clinical features/ deformities commonly found on the arms/hands of patients:
Name one sign of this illness seen on xrays:
What is the one joint never affected by this illness:
What marker is used to monitor progression of this illness:

A
Anti - CCP
ulnar deviation, boutonnieres, swan , X
Loss of joint space, soft tissue swelling, bony erosions
DIPJ
CRP
272
Q

Mary is 34 y.o. Woman who has been trying to conceive for the last 10 years. However she is always unsuccessful as the last four times she was pregnant, she ended up having a miscarriage. She and her partner have had fertility consultation. Her ovaries, fallopian tube and uterus were all normal and she has no history of STD. His sperm count was also normal. When further questioned, Mary rememebered that her mom did also tend to have miscarriages a lot and so the doctor believes this could be autoimmune caused.
What is the most likely diagnosis?

A

Antiphospholipid syndrome - stops the egg implanting and inhibts grow of foetal cells also thrombosis

273
Q

Patient has been diagnosed with systemic lupus erythematosus (SLE).
List one risk factor for this illness:
List two clinical manifestations of this illness:
What are two gold standard biochemical marker used to diagnose this illness:
Describe the ESR and CRP changes caused by this illness:
What is the first line management for this illness:

A

FH, idiopathic, Epstien barr, hydralazine, isoniazid

Malar rash , symmetrical small joint arthralgia, renal glomerulonephritis, lungs pleural effusion, heart pericarditis , cns seizures and psychosis

anti dsDNA - specific anti ANA - sensitive low speecific

Normal CRP high ESR

Oral corticosteroids ( pred ) and high dose corticosteorids ( methylpred)

274
Q

Risks for gout?

A

Thiazides, s, stopping an d starting allopurinonal surgeyr, trauma, red meats and seafood, alcohol ( beer), dehydration

275
Q

What do you see in X rays for osteoarthritis ?

A

Loss of joint space
Osteophytes
Subchodnral sclerois
Subchondral cysts

276
Q

What do you see in RE?

A

Loss of joint space
Erosions of bone
Soft bones - osteopenia
Soft tissue swelling

277
Q

What are risk factors for back pain and management?

A
Trauma and thoracic pain
Unexplained weight loss
Neurological findings/ nocturnal pain
Age over 55 or under 25
Fever
Immunocomporsed
Steroid
History of cancer or TB

E.g. mets causing spinal cord compression

Nsaids are 1 codeine second
exercsie and physio thirst

278
Q

What do you prescribe for osteoporosis?

A

Bisphosphonates ( flu like symptoms, hypocalcamiea ) and V.D and calcium

279
Q

8 year old boy has waddling gait, underweight malnourshied, wrist bony swellings,

A

Rickets - excessive non mineralised osteoid

280
Q

What is black water fever ?

A

A rare complication of anaemia where RBC haemolysis leads to dark urine

281
Q

What chromosomal abnormality is associated with multiple myeloma?

A

t11:14

282
Q
  1. You see a 50-year-old male in A&E who has presented with breathlessness, bone pain and a severe infection.
    His temperature is 39.1, his heart rate is 110 and his blood pressure is 130/85. He tells you this is the 3rd
    severe infection he has had in 6 months and thinks they are causing him to lose weight. On examination, you
    note hepatosplenomegaly and see some gum hypertrophy.
    a) What is the most likely diagnosis for this man? (1 mark)
    b) What would you expect to see on a bone marrow biopsy? (1 mark)
    c) What 2 things are associated with this cancer? (2 marks)
    d) Give 3 treatments this man is likely to receive (3 marks)
A

AML
Auer rods
Downs syndrome and radiatoin

treat: Transfuse, allopurinal, IV antibiocts, chemotherpay, steroidfs, transplant

283
Q
  1. You are a medical student on placement where you see a patient with diagnosed chronic lymphoblastic
    leukaemia. You have the opportunity to speak with him and examine him.
    a) What age would you expect this man to be? (1 mark)
    b) What would you expect to find on examination? (2 marks)
    c) What would you expect to see on a blood film? (1 mark)
    d) What treatment would this man be receiving? (2 marks)
    e) What is a complication of CLL that you should be aware of? (1 mark)
A

Over 60
Lymphadenopathy, - enlarged rubbery non tender, sweating, anorexia splenomehaly
Smudge cells
Rituximab, and bruton kinase inhibtors e.g. ibrutinib chemotherpay, allopurinaol
Richters transformatoin to aggressive lymphoma

284
Q
  1. You see a 17-year-old male with fever and night sweats. He admits to sometimes drinking with his friends
    and tells you that over the last 2 months he gets painful lumps in his neck and armpits when he drinks. He
    has no significant family history, does not smoke or take any recreational drugs.
    a) What investigations would you want to order before making a diagnosis and what signs would you
    expect to see reported in the results? (6 marks)
    b) What is the most likely diagnosis for this patient? (1 mark)
    c) What would you be the treatment plan for this patient? (3 marks)
A

FBC: Anaemia, thrombocytopenia high ESR
CXR- wide mediastinum
Blood film : Reed sternberg cells !!

Hodkins

Chemotherpay ABVD
Marrow transplant

285
Q

. A 35-year-old male presents with increased urinary frequency, pain on urination and mild back pain. He is
pyrexic, tachycardic and tachypnoeic. You send a urine sample which returns positive for a UTI. You explain
the diagnosis to the man, and tell him that he will need antibiotics to get rid of the infection. At this point, he
tells you that he has G6PD deficiency.
a) What is the function of G6PD? (2 marks)
b) What are 3 of the common symptoms of G6PD deficiency? (3 marks)
c) What would you see on a blood film for this man? (2 marks)
d) What antibiotic is now contraindicated for his UTI? (1 mark)

A

Protects RBC against oxidative stress

Fatigue, SOB< pallor and palpatations

Bite cells and reticulocytes

Cant give nitrofurantoin

286
Q

An 80-year-old man recently had a fall which resulted in a fracture neck of femur. He successfully underwent
a total hip replacement and is currently being warded in the NGH. During the ward rounds, being a good F1
doctor, you assess and examine him. It is noted that he has a swell in his left calf only. It appears slightly pale
and it is painful when you squeeze it.
a) What is the most likely diagnosis? (1 mark)
b) Pain and pallor are 2 out of the 6 symptoms for acute limb ischaemia. What are the 4 other symptoms?
(4 marks) (HINT 6 P’s)
c) Name 1 possible complication. (1 mark)
d) What is the first line investigation? (1 mark)
e) What is the name of the Scoring system used in making a diagnosis? (1 mark)
f) List 4 factors in the scoring system. (4 marks

A

DVT

Pulselessness, pallor, paresthesia , , paralysis, pershingly cold

PE
D- Dimer if positive do a gold standard US

Wells score
- Clinical features 
- HR
-Immobilization
-Previous PE or DVT 
Hemoptysis
Malignancy 
over 4 likley
287
Q

You took some bloods as part of SEPSIS 6, and the results are shown below.
• Platelet Count: Low
• Prothrombin Time: Elevated
• APTT: Elevated
• Bleeding Time: Elevated
• Fibrin Degradation Products: High D-Dimer: Elevated
• Fibrinogen levels: Low Coagulation Factor Levels: Low
a) What haematological condition does this patient have? (1 mark)
b) Explain its pathophysiology (3 marks)
c) What does a sepsis 6 involve? (6 marks) (HINT – Give 3, take 3)
d) Which antibiotic is to be prescribed for the complicated UTI caused by E. coli? (1 mark)

A

Disseminated intravascular coagulation

Platlets are acitvated leading to microthrombi NO Tissue damage causes the release and and activation of tissue factor this leads to widespread clot formation and platlet consumption and coagulation factors - increased plasminogen leads to more fibronlysis clotting and breathing

Tissue factor release coagulation - intrisnic and extrinsicn thrombosis which are fibrinolysed causing bleeding

Give flood, antibiocts, oxygeb, take urine output , cultures and LDH

Trimethoprim or NFT

288
Q

What do you see JAK -2 mutation in?

A

Polycythaemia ruby vera

Acute dehydartion, obesity, HTN . alcohol

289
Q

. Psoriasis is a systemic, immune-mediated, inflammatory skin disease that can be associated with arthritis
(psoriatic arthritis).
Name 2 ‘hidden’ places where psoriasis can be found on the body. (2 marks)

A

Behind the ear, inside the ear, nails, onokylysis, scalp, umbilicus, genitals, soles of the feet

290
Q

A patient presents to A&E with a hot, swollen, tender, and restricted knee joint and a high fever. They are
diagnosed with septic arthritis and their knee joint is urgently aspirated
List 5 risk factors for septic arthritis. (5 marks)

A
Over 80
Pre existing RA
DM
Immunosuppresion
Joint surgery 
Prosthetic joiny
Recent intraarticualr injections
Trauma
291
Q

A 73-year-old lady with previously diagnosed cancer starts to experience unremitting bone pain, which has
begun to wake her up at night. CT scans reveal bone metastases in her spine, rib, and pelvis.
What primary cancer might this patient have? Suggest two. (2 marks)

A
Breast 
Lung 
Kidney 
Thyroid 
Prostate
292
Q

SLE: Multisystem inflammator disorder name two autoantibodeis

A

ANA and antidsDNA and antismith

293
Q

Stat drug classes for osteoporsis ?

A

Bisphosphonates e.g. alendronic acid, zolendronate

taken once a week on an empty stomach must be taken standing up for hald na hour

294
Q

Where do heberdens and bouchards occur?

A

Heberdens at the distal interphalnageal joint

Bouchards at the proximal interphalangeal joint

295
Q

What’s a volvulus

A

Torsion of the colon on its mesenteric axis which contains vasculature , lymphatics and nervous supply to the bowel
Results in comprised blood flow and closed loop obstruction
Coffee bean sign
Significant sigmoidoscopy and rectal tube
Colicky pain, Abdomianl pain and absolute constipation

296
Q

What’s the portal triad?

A

Hepatic artery, portal vein and bile duct

297
Q

What’s the functional unit of the liver?

A

Sinusoid - Kupffer and stellate cells ( scarring )

298
Q

Hep B serology

A

HBsAg : Found in acute infection prescience of 6 months implies chornic
HbeAG: Found in acute infection persistence implies viral active replication and can be used to distinguish between active and inactive chronic infections
HBV- DNA: Implies viral replication in acute and chronic
Anti HBS: Immunity through immunisation or cleared infection
Anti HBC - previous or current infection
Anti IgM: Infection within 6 months
IgG : Long term
Anti HbE; Seroconversion for life

299
Q

Where do you see lead pipe?

A

UC - pseudopolyps

C

300
Q

Digestion and lipid absorption?

A

Orlistat is a lipase inhibitor

Small novel disease - bile salt malabsorption

301
Q

Itchy skin, dark urine , pale stool?

A

Cholangiocarcinoma of the pancreatic head

Pre hepatic is haemolysis e.g. malaria or H.anaemia or sepsis, red cell breakdown produces biliribun faster then the liver can take to up but its unconjugated - normal urine

Hepatic: Gilberts, viral hep, paracetomal - dark urine

Post hepatic: Obstruction e.g. gallstones , cholangiocarcinoma etc Dark urine and pale stool

302
Q

52 year old female abdo pain and vomiting gallstones and asthma, icteric sclera and tender in RUQ>

A

Ascending cholangitis - bile synthesised in liver and stored in the gall bladder

Gallstones: Cholesterol or pigment stones in gallbladder
Biliary colic: temporary obstruction of the cystic duct: intermittent epigastric /RUQ pain after pain
Acute cholecystitis: Obstruction of the gallbladder - inflammation - persistent pain and murphys sign ( can have an a calculus cholecystitis)
Ascending cholangitis: Infection of the biliary tract - gallstones, ERCP. Cholangiocarcinoma : Charcots triad of jaundice , fever, pain and RUQ
Acute pancreatitis: GET SMASHED , sever epigastric pain, vomiting, low grade fever, cullens and grey turners sign

303
Q

Severe generalised abdomianl pain, N and V, irregularly irregular pulse, ABG shows lactate and acidosis

A

Acute mesenteric ischaemia

Causes by thrombus , embolism , non occlusive ( Shock0, venous or artieral embolism
CT with contrast

Coeliac Trunk
SMA- midgut ( halfway duodenum to 2/3 across transverse colon ) - ileocolic, right colic , midcolic,
IMA: to superior 1/3 of rectum - left colic, marginal artery , sigmoid , superior rectal and mid ad inferior are from internal iliac !

304
Q

What do you find on coeliac disease?

A

Biopsy: Crypt hyperplasia
Villus atrophy
Increased intraepithelial lymphocytes
Lamina propria lymphocyte infiltration

305
Q

DIverticulitis

A

Risks: Diverticular disease , smoking , obese
Sigmoid colon usually and sometimes ascending

Signs:
Abdo pain
N and V
Fever
Change in bowel habit 
Blood is rare 

CT

ABx+ Fluids + analgesics

Surgery !

306
Q

Upper GI bleed?

A

Varicose - vomit red blood
Mallory wise -LOS
Ulcer - duodenal to gastro duodenal and gastric - left gastric

Haematemesis
Melena
Long capillary refil

Regular IBS,IV acces take blood - g + s and cross match, rests
Blatchford
Stop NSAIDS> aspirin, warfarin 
Arrange upper GI Endosocpy,y 
NBM 
Rockfall score post endosocpy
307
Q

Bile Physiology?

A

Bile primary salts is formed from cholesterol using 7 alpha hydroxylation - colic acid and chenodeoxycholic acid which are secreted to gut where they are converted to deoxycolic and lithocholic acid via bacteria conjugated by glycine and taurine - so you get 8 conjugated bile salts , CCK contracts gall bladder - 95% reabsorbed

308
Q

Which cranial nerves are involved in swallowing

A

Vagus
Soft palatable blocks nasophyranic
Epiglottis blocks await

Red: Weigjt loss, anorexia, anaemia, rapidly progression -palpable mass

Achalasia, benign stricture, malignancy, pharyngeal pouch, MG

309
Q

LFTs?

A

ALT: Liver specific
ALP: Bile duct specific , raise in bone e.g. mets and pagets
AST: Alcohol specific
GGT: Differentiates between bone and liver
Albumin
PT/INR

310
Q

Portal hTN?

A

Hypersplenism - moderate anaemia, neutropenia, thrombocytopenia
Marked ascites, capital Medusa
OEsophegeal varicose

311
Q

B12 absorption

A

Cobalamin is water soluble- DNA synthesis, myelin synthesis and RBC synthesis
Intrinsic factor is needed and produced by gastric parietal cells
Most common is pernicious anaemia
- gloss it’s, leatherargy and meagolobastic Macrocytic
IM b12 - hypersegement neurtrophls
Progressive symmetrical poly neuropathy

312
Q

Which signs are associated with acute appendicitis?

A

Rosvigs - press on LIF and you feel pain on the right
Psoas - hand on thigh and push it up against resistance - flex psoas - pain
Obturator - medial appendix - flex hip and knee and internally rotate - uses obturatro- pain
Mcburney’s: pressing on mc Burney rebound exhibits pain

Murphys: Cholecytitis

Pain in umbilicus migrating
Low grade fever, constipation/ diarrhoea
N and V

WCC+ CRP raised
Pregnancy in girls - ectopic
USS/CT
Mitselshemrt pain

When your symptoyms get better - perforated appendix !

313
Q

Which acts on parietal cells to control stomach acid secretion ?

A

Histamine - ECL
ACH - PNS acts by vagus receptors and ECL cells for histamine
Gastrin - g cells increases calcium and stimulates ECL
Somatostatin - delta cells , inhibits histamine and gastrin
Gastric acid is stimulated by gastrin, ACH, histamine and is inhibited by somatostatin

CCK is released from enteroendocrine in response to fatty chyme and binds to delta cells to release somatastin and reduce acid

314
Q

Distended abdomen, vomiting, constipation and colicky abdominal pain which imaging technique would you do?

A

CT abdomen

Small bowel obstruction

315
Q

What is the lateral border of the antecubital fossa?

A

Brachioradialis

Blood is taken from antecubital vein !

Lateral border: Brachioradialis ( extensor )
Medial ( flexors of medial epicondyle ) e.g. pronator there’s
Superior border is epicondyle line
Biceps tendon is inside
Brachial artery and vein
Median and radial nerve
Ulnar is around medial epicondyle

316
Q

What does the long thoracic nerve innervated?

A

Serratus anterior - winged scapula
C5,C6,C7

Main deep: Latissamus Doris and trapezius
Then deep t this: Rhomboid - retraction of scapula
Roatation cuff: Supraspiantus, infraspinatus, teres minor, teres major isn’t !
Sub scapularis is the 4
Deltoid is on top

317
Q

What does the head of the radius Articular with

A

Capitulum of the humerus:

Head and ridge - anatomical neck but the surgical neck is below - higher fracture chance
Greater tubercle - attaches rotator cuffs
Biceptial groove between greater and lesser tubuercule for the long head of the biceps
Deltoid tuberosity for deltoid
Posterior radial/ spiral groove for radial nerve
Condyles and lateral is capitulum and the lumps are the trochleor
Head of the radius articulates with capitulum
Ulnar arituckakr with the trochleor - olecranon porcess articulates into olecranono fossa , head of ulnar is near the wrists
Epicondyles medialandlateral

318
Q

What do you find in the popliteal fossa?

A
Popliteal nerve
Poliptal artery 
Popliteal vein 
Tibial nerve
Common perineal ( fibulae nerve )

Superiomedial border - semimembranous muscle
Superior lateral border - biceps femurs
Infromedial border - medial head of gastrocnemius
Medial lateral: Lateral head of gastrocnemius and plantar is

Popliteal artery , popliteal vein, tibial nerve and common perineal nerve ( branches of sciatic)
Short saphenous vein passes between heads of gastoneaemus
Can get bakers cysts

319
Q

What does cutaeous innervation to the index finger?

A

Median and radial nerve

Ulnar does pinkie and half of the ring finger
Median does - From middle of the ring finger - anterior is median nerve and nail beds
Radial does posterior part of muffle of ring finger and hand of other 3 fingers

320
Q

Hand muscles

A

Superficial: Pronator teres, Flexor carpi radialias,palmaris longus, flexor carpi ulnaris

deep: flexor digitorrium profundus, flexor polllicus longus and pronator quadrate s

321
Q

Numbness over right shoulder and restricted movement, cant abduct beyond 15 degrees, loss inferior deltoid

A

Axillary: Semsory regimental badge and does teres minor and deltoid

Exits at subscalaris at quandarangular space - compression risk and posterior circumflaex -lateral cutanus nerve

322
Q

What’s the medial border of the femoral triangle ?

A

Top: Inguinal ligament, lateral is sartorius

Adductor longus !
Sartorius is lateral

contains femoral nerve , common femoral artery, femoral vein gives of great saphinous

Flood : Ileopsoa is floor
NAVI

323
Q

A 31 year old female has tingling in her thumb index and middle fingers which gets worse at night, Sympotyms get worse when she holds her wrist in flexion whats been affected?

A

Median nerve - CTS
Compression of the median nerve within the carpal tunnel - pain numbness and parastehsia in the lateral 3 digits
Female, increasing age, pregnancy and obesity , DM, RA, hypothyroidism , repetitive movements
Tinels and phalens
Clinical diagnosis or nerve conduction studies
Management - splint and corticosteroid injections
Shake arm
Carpal tunnel decompression
Palm is spared

324
Q

Mid shaft humeral fracture what would present?

A

Wrist drop

Surgical neck - nerve axillary gets damaged - deltoid and rotator cuff weakness of shoulder abduction
Shaft: Spiral radial nerve groove - wrist drop - triceps and extensors
Claw Hand: ulnar nerve - change extend
Sign of benediction: median nerve
Musculocutanus: Weakness of elbow flexion: Biceps brachii, brachialis and Cora o brachialis

Ok sign
Spread fingers

325
Q

Swelling and when knee is flexed at 90 degrees you get significant tibial excursion when force is applied in the anterior direction

A

Anterior crucial efforts ligament

Patellar ligament is a continuation of the quadriceps femoris
Lateral and medial collateral - Prevnts excessive medial or lateral movement
Medial : Medial epicondyle of femur and medial condole of the tibia
LCL: Lateral epicondyle and depression on the fibula head
Cruciate ligaments connect femur and Tibia - anterior cruciate attaches at anterior intercondyle region
Blends with medial meniscus and ascends posterior Ely to attach to the femur at the intercondyle fossa and prevents anterior dislocation
Posterior cruciate: Posterior intercondyle to anterior medial femoral condole ( prevents posterior dislocation of the tibia onto the femur:

AC injury:
Twisting knee whilst weight baring no contact
Rapid joint swelling , pain and instability
Lachmans test and anterior draw test
X ray excludes bony injuries and MRI
Manages: RICE and Rehab
Tendon or graft
Can get bony emulsion of the lateral proximal tibia - segulds fracture
MRI is gold

326
Q

A tree surgeon falls with his harm in a full abducted position above the head what would you see?

A

Claw hand

Long thoracic is adherent to wall of the thorax

Klumpers palsy - claw hand ulnar nerve

327
Q

Brachial Plexus?

A

Upper part : Erbs palsy e.g. traumatic birth waiters tip - pronated internally rotate cant flex elbow
C5-T1
RTDCB

Musculocutaneus : Biceps brachii, brachialis an coracobrachialis and skin overlying
median and ulnar supply below elbow
Median - most of anterior foramen and ulnar is hand but median does LOAF

328
Q

Pain in elbow after painting worse carrying heavy objects improves on rest its recreated with wrist extension and forearm supination with an extended elbow and maximal pain is never the insertion of the common extensor tendon whats caused it ?

A

Lateral Epicondylitis and medial epicondylitis
Repetitive overused - microtears -fibrosis - tendinosis
Pain affecting elbow radiating down forearm
Examination: Local tenderness, cozens and mikes
CLinical or USS and MRI
Modify activity analgesia, corticosteroid injection

329
Q

Pain in right hip, leg is shortened and internally rotated which artery’s branches supply the had of the femur?

A

Femoral - Profundus femoris !

Fetus is obtruator artery - obdurate ligament

330
Q

Breaking of femur neck?

A

High risk of avascular necrosis

331
Q

Fell on wrist and pain on movement and tender in anatomical snuff box what has je fracture?

A

Scaphoid - retrograde blood supply
Blood supply

Snuffbox: Radial arter, nerve and cephalic vein floor is the scaphoid
X ray may need to do 2 10 days apart
Immobilise
MRI is most definitive

332
Q

What do you see on X ray for osteoarthritis

A

Loss of joint space
Osteophytea t joint margins
Subchondral cysts
Subchondral sclerosis

RA: Lojs, periaticualr erosions, osteopenia and soft tissue swelling